HA Exam 4

अब Quizwiz के साथ अपने होमवर्क और परीक्षाओं को एस करें!

Completion: During the assessment of deep tendon reflexes, the nurse finds that a patient's responses are bilaterally normal. What number is used to indicate normal deep tendon reflexes when the documenting this finding? ____+Correct.

ANS: 2 Responses to assessment of deep tendon reflexes are graded on a 4-point scale. A rating of 2+ indicates normal or average response. A rating of 0 indicates no response, and a rating of 4+ indicates very brisk, hyperactive response with clonus, which is indicative of disease.

A patient has been diagnosed with osteoporosis and asks the nurse, "What is osteoporosis?" What is the best explanation by the nurse? a. "It is the loss of bone density." b. "It is an increase in bone matrix." c. "It is new bone growth that is weaker." d. "There is a decrease in phagocytic activity."

ANS: A After age 40 years, a loss of bone matrix (resorption) occurs more rapidly than new bone formation. The net effect is a gradual loss of bone density, or osteoporosis. The other options are not correct. There is a decrease, not increase, in bone matrix with aging; new bone growth is slower than the loss of bone matrix (not weaker bone growth); and phagocytic activity has nothing to do with bones.

START OF BREASTS, AXILLAE, AND REGIONAL LYMPHATICS QUESTIONS: Which of the following statements is true regarding the internal structures of the breast? a. Fibrous, glandular, and adipose tissues b. Primarily muscle with very little fibrous tissue c. Primarily milk ducts, known as lactiferous ducts d. Glandular tissue, which supports the breast by attaching to the chest wall

ANS: A The breast is made up of glandular, fibrous (including the suspensory ligaments), and adipose tissues.

The nurse is reviewing a patient's medical record and notes that he is in a coma. Using the Glasgow Coma Scale, which number indicates that the patient is in a coma? a. 6 b. 12 c. 15 d. 24

ANS: A A fully alert, normal person has a score of 15, whereas a score of 7 or less reflects coma on the Glasgow Coma Scale

In the assessment of a 1-month-old infant, the nurse notices a lack of response to noise or stimulation. The mother reports that in the last week he has been sleeping all of the time, and when he is awake all he does is cry. The nurse hears that the infant's cries are very high pitched and shrill. What is the most appropriate response by the nurse? a. Refer the infant for further testing. b. Talk with the mother about eating habits. c. Do nothing; these are expected findings for an infant this age. d. Tell the mother to bring the baby back in 1 week for a recheck.

ANS: A A high-pitched, shrill cry or cat-sounding screech occurs with central nervous system damage. Lethargy, hyporeactivity, and hyperirritability, as well as the parent's report of significant changes in behavior all warrant referral. The other options are not correct responses.

A 43-year-old woman is at the clinic for a routine examination. She reports that she has had a lump in her right breast for years. Recently, it has begun to change in consistency and is becoming harder. She reports that 5 years ago her physician evaluated the lump and determined that it was "nothing to worry about." The nurse's examination validates the presence of a mass in the right upper outer quadrant at 1 o'clock, approximately 5 cm from the nipple. It is firm, mobile, and nontender, with borders that are not well defined. What is the best response by the nurse? a. "Because of the change in consistency of the lump, it should be further evaluated by a physician." b. "The changes could be r/t your menstrual cycles. Keep track of the changes in the mass each month." c. "The lump is probably nothing to worry about because it has been present for years and was determined to be noncancerous 5 years ago." d. "Because you are experiencing no pain and the size has not changed, you should continue to monitor the lump and return to the clinic in 3 months."

ANS: A A lump that has been present for years and is not exhibiting changes may not be serious but should still be explored. Any recent change or a new lump should be evaluated. The other responses are not correct.

A 21-year-old patient has a head injury resulting from trauma and is unconscious. There are no other injuries. During the assessment what would the nurse expect to find when testing the patient's deep tendon reflexes? a. Reflexes will be normal. b. Reflexes cannot be elicited. c. All reflexes will be diminished but present. d. Some reflexes will be present, depending on the area of injury

ANS: A A reflex is a defense mechanism of the nervous system. It operates below the level of conscious control and permits a quick reaction to potentially painful or damaging situations.

During an examination, the nurse notes a supernumerary nipple just under the patient's left breast. The patient tells the nurse that she always thought it was a mole. Which statement about this finding is correct? a. This variation is normal and not a significant finding. b. This finding is significant and needs further investigation. c. A supernumerary nipple also contains glandular tissue and may leak milk during pregnancy and lactation. d. The patient is correct—a supernumerary nipple is actually a mole that happens to be located under the breast.

ANS: A A supernumerary nipple looks like a mole, but close examination reveals a tiny nipple and areola; it is not a significant finding. This is not a significant finding and does not need further investigation, will not leak milk during pregnancy and lactation, and is not a mole. Although a supernumerary nipple looks like a mole, upon close examination a tiny nipple and areola are revealed.

In obtaining a health history on a 74-year-old patient, the nurse notes that he drinks alcohol daily and that he has noticed a tremor in his hands that affects his ability to hold things. With this information, what response should the nurse make? a. "Does the tremor change when you drink alcohol?" b. "Does your family know you are drinking every day?" c. "We'll do some tests to see what is causing the tremor." d. "You really shouldn't drink so much alcohol; it may be causing your tremor."

ANS: A Although not a recommended treatment, senile tremor is relieved by alcohol. The nurse should assess how alcohol affects the tremor and whether the person is abusing alcohol in an effort to relieve the tremor. Asking whether the family knows he drinks daily does not address the issue of the tremor and it is possible cause. Before ordering tests, a thorough assessment should be performed. Telling the patient he shouldn't drink so much and that drinking may be the cause of his tremor is inappropriate and will likely make the patient defensive.

During an annual physical examination, a 43-year-old patient states that she does not perform monthly breast self-examinations (BSEs). She tells the nurse that she believes that mammograms "do a much better job than I ever could to find a lump." What should the nurse include in his or her response to this patient? a. BSEs may detect lumps that appear between mammograms. b. She is correct—mammography is a good replacement for BSE. c. The American Cancer Society recommends women over 40 years old perform a monthly BSE. d. She does not need to perform BSEs as long as a physician checks her breasts annually.

ANS: A Although the American Cancer Society no longer recommends a structured monthly BSE because many women with breast cancer have detected their lumps by chance as when bathing or dressing, the goal of BSE is that a woman becomes familiar with the look and feel of her breasts so she can detect any change and report it promptly. Intermittent BSEs along with clinical breast examinations and mammograms are complementary screening measures. Mammography can reveal cancers too small to be detected by the woman or by the most experienced examiner. However, interval lumps may become palpable between mammograms

To assess the head control of a 4-month-old infant, the nurse lifts up the infant in a prone position while supporting his chest. The nurse looks for what normal response? a. Infant raises the head and arches the back. b. Infant extends the arms and drops down the head. c. Infant flexes the knees and elbows with the back straight. d. Infant holds the head at 45 degrees and keeps the back straight.

ANS: A At 3 months of age, the infant raises the head and arches the back as if in a swan dive. This response is the Landau reflex, which persists until 11 2 years of age. The other responses are incorrect.

During a speculum inspection of the vagina, what would the nurse expect to see at the end of the vaginal canal? a. Cervix b. Uterus c. Ovaries d. Fallopian tubes

ANS: A At the end of the canal, the uterine cervix projects into the vagina

A 40-year-old man has come into the clinic reporting extreme pain in his toes. The nurse notices that his toes are slightly swollen, reddened, and warm to the touch. What does the nurse suspect? a. Acute gout b. Osteoporosis c. Ankylosing spondylitis d. Degenerative joint disease

ANS: A Clinical findings for acute gout consist of redness, swelling, heat, and extreme pain like a continuous throbbing. Gout is a metabolic disorder of disturbed purine metabolism, associated with elevated serum uric acid. Osteoporosis is a decrease in skeletal bone mass leading to low bone mineral density and impaired bone density which increases the risk for fractures. It occurs primarily in postmenopausal white women. Ankylosing spondylitis is chronic inflamed vertebrae and is characterized by inflammatory back pain that is dull and deep in lower back or buttocks. Degenerative joint disease (osteoarthritis) is a localized, progressive disorder involving deterioration of articular cartilages and subchondral bone remodeling, synovial inflammation, and formation of new bone at joint surfaces. Asymmetric joint involvement commonly affects hands, knees, hips, and lumbar and cervical segments of the spine. This patient's symptoms are consistent with acute gout.

A 59-year-old patient has been diagnosed with prostatitis and is being seen at the clinic for reports of burning and pain during urination. How should the nurse document this finding? a. Dysuria b. Nocturia c. Polyuria d. Hematuria

ANS: A Dysuria (burning with urination) is common with acute cystitis, prostatitis, and urethritis. Nocturia is voiding during the night. Polyuria is voiding in excessive quantities. Hematuria is voiding with blood in the urine.

The nurse is examining a 6-month-old infant and places the infant's feet flat on the table and flexes his knees up. The nurse notes that the right knee is significantly lower than the left. Which of these statements is true of this finding? a. This finding is a positive Allis sign and suggests hip dislocation. b. The infant probably has a dislocated patella on the right knee. c. This finding is a negative Allis sign and normal for an infant of this age. d. The infant should return to the clinic in 2 weeks to see if his condition has changed.

ANS: A Finding one knee significantly lower than the other is a positive Allis sign and suggests hip dislocation. Normally the tops of the knees are at the same elevation. The other statements are not correct.

A patient tells the nurse that she is having a hard time bringing her hand to her mouth when she eats or tries to brush her teeth. The nurse knows that for her to move her hand to her mouth, she must perform which movement? a. Flexion b. Abduction c. Adduction d. Extension

ANS: A Flexion, or bending a limb at a joint, is required to move the hand to the mouth. Extension is straightening a limb at a joint. Moving a limb toward the midline of the body is called adduction; abduction is moving a limb away from the midline of the body.

In a person with an upper motor neuron lesion such as a cerebrovascular accident, which of these physical assessment findings should the nurse expect? a. Hyperreflexia b. Fasciculations c. Loss of muscle tone and flaccidity d. Atrophy and wasting of the muscles

ANS: A Hyperreflexia, diminished or absent superficial reflexes, and increased muscle tone or spasticity can be expected with upper motor neuron lesions. The other options reflect a lesion of lower motor neurons.

The nurse has palpated a lump in a female patient's right breast. The nurse documents this as a small, round, firm, distinct, lump located at 2 o'clock, 2 cm from the nipple. It is nontender and fixed. No associated retraction of the skin or nipple, no erythema, and no axillary lymphadenopathy are observed. What information is missing from the documentation? a. Size of the lump b. Shape of the lump c. Consistency of the lump d. Whether the lump is solitary or multiple

ANS: A If the nurse feels a lump or mass, then he or she should note these characteristics: (1) location, (2) size judge in centimeters in three dimensions: width × length × thickness, (3) shape, (4) consistency, (5) motility, (6) distinctness, (7) nipple, (8) the skin over the lump, (9) tenderness, and (10) lymphadenopathy.

The mother of a 5-year-old girl tells the nurse that she has noticed her daughter "scratching at her bottom a lot the last few days." During the assessment, the nurse finds redness and raised skin in the anal area. What does this finding likely indicate? a. Pinworms b. Chickenpox c. Constipation d. Bacterial infection

ANS: A In children, pinworms are a common cause of intense itching and irritated anal skin. The other options are not correct.

When assessing a newborn infant's genitalia, the nurse notices that the genitalia are somewhat engorged. The labia majora are swollen, the clitoris looks large, and the hymen is thick. The vaginal opening is difficult to visualize. The infant's mother states that she is worried about the labia being swollen. How should the nurse reply? a. "This is a normal finding in newborns and should resolve within a few weeks." b. "This finding could indicate an abnormality and may need to be evaluated by a physician." c. "We will need to have estrogen levels evaluated to ensure that they are within normal limits." d. "We will need to keep close watch over the next few days to see if the genitalia decrease in size."

ANS: A It is normal for a newborn's genitalia to be somewhat engorged. A sanguineous vaginal discharge or leukorrhea is normal during the first few weeks because of the maternal estrogen effect. During the early weeks, the genital engorgement resolves, and the labia minora atrophy and remain small until puberty.

During an examination, which tests will the nurse collect to screen for cervical cancer? a. Endocervical specimen, cervical scrape, and vaginal pool b. Endocervical specimen, vaginal pool, and acetic acid wash c. Cervical scrape, acetic acid wash, saline mount (wet prep) d. Endocervical specimen, potassium hydroxide (KOH) preparation, and acetic acid wash

ANS: A Laboratories may vary in method, but usually the test consists of three specimens: endocervical specimen, cervical scrape, and vaginal pool. The other tests (acetic acid wash, KOH preparation, and saline mount) are used to test for sexually transmitted infections.

A 54-year-old man comes to the clinic with a "horrible problem." He tells the nurse that he has just discovered a lump on his breast and is fearful of cancer. The nurse knows which statement about breast cancer in men is true? a. Breast cancer in men rarely spreads to the lymph nodes. b. Less than one percent of all breast cancers occurs in men. c. Most breast masses in men are diagnosed as gynecomastia. d. Breast masses in men are difficult to detect because of minimal breast tissue

ANS: A Less than one percent of all breast cancers occur in men. The early spreading to axillary lymph nodes is attributable to minimal breast tissue. Breast cancer in men often spreads to the axillary lymph nodes because of the minimal breast tissue in men. Breast masses in men are not typically diagnosed as gynecomastia or more difficult to detect, but they are diagnosed 10 years later than women because of lack of screening and general awareness. The correct option is that less than one percent of all breast cancers occur in men.

A woman who is 8 months pregnant comments that she has noticed a change in her posture and is having lower back pain. The nurse tells her that during pregnancy, women have a posture shift to compensate for the enlarging fetus. What is the term for this shift in posture? a. Lordosis b. Scoliosis c. Ankylosis d. Kyphosis

ANS: A Lordosis compensates for the enlarging fetus, which would shift the center of balance forward. This shift in balance, in turn, creates a strain on the low back muscles, felt as low back pain during late pregnancy by some women. Scoliosis is lateral curvature of portions of the spine; ankylosis is extreme flexion of the wrist, as observed with severe rheumatoid arthritis; and kyphosis is an enhanced thoracic curvature of the spine. Scoliosis is lateral curvature of portions of the spine; ankylosis is extreme flexion of the wrist, as observed with severe rheumatoid arthritis; and kyphosis is an enhanced thoracic curvature of the spine. The symptoms this patient is experiencing are lordosis.

A woman is in the clinic for an annual gynecologic examination. How should the nurse begin the interview? a. Menstrual history, because it is generally nonthreatening. b. Sexual history, because discussing it first will build rapport. c. Obstetric history, because it includes the most important information. d. Urinary system history, because problems may develop in this area as well.

ANS: A Menstrual history is usually nonthreatening and therefore a good topic with which to begin the interview. Obstetric, urinary, and sexual histories are also part of the interview but not necessarily the best topics with which to start.

To test for gross motor skill and coordination of a 6-year-old child, which of these techniques would be appropriate? a. Have the child hop on one foot. b. Have the child stand on his head. c. Ask the child to touch his finger to his nose. d. Ask the child to make "funny" faces at the nurse.

ANS: A Normally a child can hop on one foot and balance on one foot for approximately 5 seconds by 4 years of age and can balance on one foot for 8 to 10 seconds at 5 years of age. Children enjoy performing these tests. Failure to hop after 5 years of age indicates incoordination of gross motor skills. Asking the child to touch his or her finger to the nose checks fine motor coordination; and asking the child to make "funny" faces tests CN VII. Asking a child to stand on his or her head is not appropriate

When performing a genitourinary assessment, the nurse notices that the urethral meatus is ventrally positioned. What does this indicate? a. Hypospadias b. A result of phimosis c. Probably due to a stricture d. Often associated with aging

ANS: A Normally the urethral meatus is positioned just about centrally. Hypospadias is the ventral location of the urethral meatus. The position of the meatus does not change with aging. Phimosis is the inability to retract the foreskin. A stricture is a narrow opening of the meatus.

An 85-year-old patient comments during his annual physical examination that he seems to be getting shorter as he ages. Why does height decrease with aging? a. The vertebral column shortens. b. Long bones tend to shorten with age. c. A significant loss of subcutaneous fat occurs. d. A thickening of the intervertebral disks develops.

ANS: A Postural changes are evident with aging and decreased height is most noticeable due to shortening of the vertebral column. Long bones do not shorten with age. Intervertebral disks actually get thinner with age. Subcutaneous fat is not lost but is redistributed to the abdomen and hips.

While obtaining a health history of a 3-month-old infant from the mother, the nurse asks about the infant's ability to suck and grasp the mother's finger. What is the nurse assessing? a. Reflexes b. Intelligence c. Cranial nerves d. Cerebral cortex function

ANS: A Questions regarding an infant's ability to suck and grasp are assessing the infant's reflexes. Questions regarding reflexes include such questions as, "What have you noticed about the infant's behavior," "Do the infant's sucking and swallowing seem coordinated," and "Does the infant grasp your finger?" The other responses are incorrect.

The nurse is inspecting the scrotum and testes of a 43-year-old man. Which finding would require additional follow-up and evaluation? a. Skin on the scrotum is taut. b. Left testicle hangs lower than the right testicle. c. Scrotal skin has yellowish 1-cm nodules that are firm and nontender. d. Testes move closer to the body in response to cold temperatures.

ANS: A Scrotal swelling may cause the skin to be taut and to display pitting edema. Normal scrotal skin is rugae, and asymmetry is normal with the left scrotal half usually lower than the right. The testes may move closer to the body in response to cold temperatures.

During the assessment of an 80-year-old patient, the nurse notices that his hands show tremors when he reaches for something and his head is always nodding. No associated rigidity is observed with movement. Which of these statements is most accurate? a. These findings are normal, resulting from aging. b. These findings could be r/t hyperthyroidism. c. These findings are the result of Parkinson disease. d. This patient should be evaluated for a cerebellar lesion.

ANS: A Senile tremors occasionally occur. These benign tremors include an intention tremor of the hands, head nodding (as if saying yes or no), and tongue protrusion. Tremors associated with Parkinson disease include rigidity, slowness, and a weakness of voluntary movement. The other responses are incorrect.

When performing an external genitalia examination of a 10-year-old girl, the nurse notices that no pubic hair has grown in and the mons and the labia are covered with fine vellus hair. According to the Sexual Maturity Rating scale, what stage of sexual maturity do these findings indicate? a. 1 b. 2 c. 3 d. 4

ANS: A Sexual Maturity Rating stage 1 is the preadolescent stage. There is no pubic hair, and the mons and labia are covered with fine, vellus hair as on the abdomen. In stage 2 hair growth is sparse and mostly on the labia; long, downy hair, slightly pigmented, straight or only slightly curly. In stage 3 hair growth is sparse and spreading over mons pubis. Hair is darker, coarser, and curlier. In stage 4 hair is adult in type but over smaller area, none on medial thigh.

The nurse is testing the deep tendon reflexes of a 30-year-old woman who is in the clinic for an annual physical examination. When striking the Achilles heel and quadriceps muscle, the nurse is unable to elicit a reflex. How should the nurse proceed? a. Ask the patient to lock her fingers and pull. b. Document these reflexes as 0 on a scale of 0 to 4+. c. Refer the patient to a specialist for further testing. d. Complete the examination, and then test these reflexes again

ANS: A Sometimes the reflex response fails to appear. Documenting the reflexes as absent is inappropriate this soon in the examination. The nurse should try to further encourage relaxation, varying the person's position or increasing the strength of the blow. Reinforcement is another technique to relax the muscles and enhance the response. The person should be asked to perform an isometric exercise in a muscle group somewhat away from the one being tested. For example, to enhance a patellar reflex, the person should be asked to lock the fingers together and pull.

A male patient with possible fertility problems asks the nurse where sperm is produced. Which answer should the nurse give the patient? a. Testes b. Prostate c. Epididymis d. Vas deferens

ANS: A Sperm production occurs in the testes, not in the other structures listed.

A 70-year-old man is visiting the clinic for difficulty in passing urine. In the health history, he indicates that he has to urinate frequently, especially at night. He has burning when he urinates and has noticed pain in his back. Considering this history, what might the nurse expect to find during the physical assessment? a. Asymmetric, hard, and fixed prostate gland b. Occult blood and perianal pain to palpation c. Symmetrically enlarged, soft prostate gland d. Soft nodule protruding from the rectal mucosa

ANS: A Subjective symptoms of carcinoma of the prostate include frequency, nocturia, hematuria, weak stream, hesitancy, pain or burning on urination, and continuous pain in lower back, pelvis, and thighs. Objective symptoms of carcinoma of the prostate include a malignant neoplasm that often starts as a single hard nodule on the posterior surface, producing asymmetry and a change in consistency. As it invades normal tissue, multiple hard nodules appear, or the entire gland feels stone hard and fixed.

An 11-year-old girl is in the clinic for a sports physical examination. The nurse notices that she has begun to develop breasts, and during the conversation the girl reveals that she is worried about her development. The nurse should use which of these techniques to best assist the young girl in understanding the expected sequence for development? a. Use the Tanner scale on the five stages of sexual development. b. Describe her development and compare it with that of other girls her age. c. Use the Jacobsen table on expected development on the basis of height and weight data. d. Reassure her that her development is within normal limits and tell her not to worry about the next step.

ANS: A The Tanner scale on the five stages of pubic hair development is helpful in teaching girls the expected sequence of sexual development (see Table 26-1). The other responses are not appropriate.

What is articulated with the tibia and fibula in the ankle joint? a. Talus b. Cuboid c. Calcaneus d. Cuneiform bones

ANS: A The ankle or tibiotalar joint is the articulation of the tibia, fibula, and talus. The other bones listed are foot bones and not part of the ankle joint.

A woman has just learned that she is pregnant. What should the nurse teach this patient about changes in her breasts? a. She can expect her areolae to become larger and darker in color. b. Breasts may begin secreting milk after the fourth month of pregnancy. c. She should inspect her breasts for visible veins and immediately report these. d. During pregnancy, breast changes are fairly uncommon; most of the changes occur after the birth.

ANS: A The areolae become larger and grow a darker brown as pregnancy progresses, and the tubercles become more prominent. (The brown color fades after lactation, but the areolae never return to their original color). A venous pattern is an expected finding and prominent over the skin surface and does not need to be reported. After the fourth month of pregnancy, colostrum, a thick, yellow fluid (precursor to milk), may be expressed from the breasts.

The nurse is caring for a newborn infant. Thirty hours after birth, the infant passes a dark green meconium stool. What is the importance of this finding? a. The stool indicates anal patency. b. The dark green color indicates occult blood in the stool. c. Meconium stool can be reflective of distress in the newborn. d. The newborn should have passed the first stool within 12 hours of birth.

ANS: A The first stool passed by the newborn is dark green meconium and occurs within 24 to 48 hours of birth, indicating anal patency. The other responses are not correct.

The wife of a 65-year-old man tells the nurse that she is concerned because she has noticed a change in her husband's personality and ability to understand. He also cries very easily and becomes angry. What part of the cerebral lobe is responsible for these behaviors? a. Frontal b. Parietal c. Occipital d. Temporal

ANS: A The frontal lobe has areas responsible for personality, behavior, emotions, and intellectual function. The parietal lobe has areas responsible for sensation; the occipital lobe is responsible for visual reception; and the temporal lobe is responsible for hearing, taste, and smell.

The nurse is examining a female patient's vestibule. What does the nurse expect to visualize? a. Urethral meatus and vaginal orifice b. Vaginal orifice and vestibular (Bartholin) glands c. Urethral meatus and paraurethral (Skene) glands d. Paraurethral (Skene) and vestibular (Bartholin) glands

ANS: A The labial structures encircle a boat-shaped space, or cleft, termed the vestibule. Within the vestibule are numerous openings. The urethral meatus and vaginal orifice are visible. The ducts of the paraurethral (Skene) glands and the vestibular (Bartholin) glands are present but not visible.

When performing a musculoskeletal assessment, what is the correct approach? a. Proximal to distal b. Distal to proximal c. Posterior to anterior d. Anterior to posterior

ANS: A The musculoskeletal assessment should be performed in an orderly approach, head to toe, proximal to distal, from the midline outward. The other options are not correct.

The nurse is assisting with a BSE clinic. Which of these women reflects abnormal findings during the inspection phase of breast examination? a. Woman whose nipples are in different planes (deviated) b. Woman whose left breast is slightly larger than her right c. Nonpregnant woman whose skin is marked with linear striae d. Pregnant woman whose breasts have a fine blue network of veins visible under the skin

ANS: A The nipples should be symmetrically placed on the same plane on the two breasts. With deviation in pointing, an underlying cancer may cause fibrosis in the mammary ducts, which pulls the nipple angle toward it. The other examples are normal findings. A woman with a left breast that is slightly larger than her right, a nonpregnant woman whose skin is marked with linear striae, and a pregnant woman whose breasts have a fine blue network of vein visible under the skin are all normal findings. However, a woman whose nipples are in different planes, or deviate, is abnormal.

The nurse is preparing to palpate the rectum and should use which of these techniques? a. Flex the finger, and slowly insert it toward the umbilicus. b. Insert an extended index finger at a right angle to the anus. c. First instruct the patient that this procedure will be painful. d. Place the finger directly into the anus to overcome the tight sphincter.

ANS: A The nurse should gently place the pad of the index finger against the anal verge. The nurse will feel the sphincter tighten and then relax. As it relaxes, the nurse should flex the tip of the finger and slowly insert it into the anal canal in a direction toward the umbilicus. The nurse should never approach the anus at right angles with the index finger extended; doing so would cause pain. The nurse should instruct the patient that palpation is not painful but may feel like needing to move the bowels.

The nurse is palpating a female patient's breasts during an examination. Which of these positions is most likely to make significant lumps more distinct during breast palpation? a. Supine with the arms raised over her head b. Sitting with the arms relaxed at her sides c. Supine with the arms relaxed at her sides d. Sitting with the arms flexed and fingertips touching her shoulders

ANS: A The nurse should help the woman to a supine position, tuck a small pad under the side to be palpated, and help the woman raise her arm over her head. These maneuvers will flatten the breast tissue and medially displace it. Any significant lumps will then feel more distinct.

Which of these statements is most appropriate when the nurse is obtaining a genitourinary history from an older man? a. "Do you need to get up at night to urinate?" b. "Do you experience nocturnal emissions, or 'wet dreams'?" c. "Do you know how to perform a testicular self-examination?" d. "Has anyone ever touched your genitals when you did not want them to?"

ANS: A The older male patient should be asked about the presence of nocturia. Awaking at night to urinate may be attributable to a diuretic medication, fluid retention from mild heart failure or varicose veins, or fluid ingestion 3 hours before bedtime, especially coffee and alcohol. The other questions are more appropriate for younger men.

A patient is newly diagnosed with benign breast disease. The nurse recognizes which statement about benign breast disease to be true? a. It makes it more difficult to examine the breasts. b. It is easily reduced with hormone replacement therapy. c. It frequently turns into cancer in a woman's later years. d. It is usually diagnosed before a woman reaches childbearing age.

ANS: A The presence of benign breast disease (formerly fibrocystic breast disease) makes it hard to examine the breasts; the general lumpiness of the breast conceals a new lump. The other statements are not true. Benign breast disease is not treated with hormone replacement therapy and it does not usually turn into cancer, although the nodularity associated with benign breast disease makes it difficult to detect other cancerous lumps. It also is usually diagnosed between 30 and 55 years old (not before childbearing age).

During a breast examination on a female patient, the nurse notices that the nipple is flat, broad, and fixed. The patient states it "started doing that a few months ago." What does this finding suggest? a. Dimpling b. Retracted nipple c. Nipple inversion d. Deviation in nipple pointing

ANS: A The retracted nipple looks flatter and broader, similar to an underlying crater. A recent retraction suggests cancer, which causes fibrosis of the whole duct system and pulls in the nipple. It also may occur with benign lesions such as ectasia of the ducts. The nurse should not confuse retraction with the normal long-standing type of nipple inversion, which has no broadening and is not fixed.

During a group discussion on men's health, what group should the nurse inform them has the highest incidence of prostate cancer? a. Blacks b. Hispanics c. Asian Americans d. American Indians

ANS: A The risk for prostate cancer is 74% higher in African-American and African-Caribbean men. Reasons are not known but may be r/t inherited genetic factors.

During an assessment of a 62-year-old man, the nurse notices the patient has a stooped posture, shuffling walk with short steps, flat facial expression, and pill-rolling finger movements. What do these findings suggest? a. Parkinsonism b. Cerebral palsy c. Cerebellar ataxia d. Muscular dystrophy

ANS: A The stooped posture, shuffling walk, short steps, flat facial expression, and pill-rolling finger movements are all found in parkinsonism. Cerebral palsy is dysfunction of a mixed group of paralytic neuromotor disorders of infancy and childhood due to damage to cerebral cortex from a developmental defect, intrauterine meningitis, encephalitis, birth trauma, anoxia, or kernicterus and may present as spasticity or athetosis. The characteristics of cerebellar ataxia include a staggering, wide-based gait; difficulty with turns; and uncoordinated movement with positive Romberg sign. Muscular dystrophy is a chronic, progressive wasting of the musculature, which produces weakness, contractures, and in severe cases respiratory dysfunction and death. Weak pelvis muscles and decreased or absent reflexes are signs of muscular dystrophy.

A new mother calls the clinic to report that part of her left breast is red, swollen, tender, very hot, and hard. She has a fever of 38.3° C. She also has had symptoms of influenza, such as chills, sweating, and feeling tired. The nurse notices that she has been breastfeeding for 1 month. From her description, what condition does the nurse suspect? a. Mastitis b. Paget disease c. Plugged milk duct d. Mammary duct ectasia

ANS: A The symptoms describe mastitis, which stems from an infection or stasis caused by a plugged duct. A plugged duct does not have infection present. Paget disease starts with a small crust on the nipple apex and then spreads to the areola. A plugged milk duct often presents as a tender lump that may be reddened and warm to touch but no infection is present. Mammary duct ectasia is caused by stagnation of cellular debris and secretions in the ducts, leading to obstruction, inflammation and infection. Itching, burning, or drawing pain occurs around the nipple with a sticky, purulent discharge that may be cream, green, or bloody. The symptoms this patient has described mastitis, which stems from an infection or stasis caused by a plugged duct. A plugged duct does not have infection present but a plugged duct with infection is mastitis

The uterus is usually positioned tilting forward and superior to the bladder. What is this position called? a. Anteverted and anteflexed b. Retroverted and anteflexed c. Retroverted and retroflexed d. Superiorverted and anteflexed

ANS: A The uterus is freely movable, not fixed, and usually tilts forward and superior to the bladder (a position labeled as anteverted and anteflexed).

The nurse is reviewing the changes that occur with menopause. Which changes are expected? a. Uterine and ovarian atrophy, along with thinning of vaginal epithelium b. Ovarian atrophy, increased vaginal secretions, and increasing clitoral size c. Cervical hypertrophy, ovarian atrophy, and increased acidity of vaginal secretions d. Vaginal mucosa fragility, increased acidity of vaginal secretions, and uterine hypertrophy

ANS: A The uterus shrinks because of its decreased myometrium. The ovaries atrophy to 1 to 2 cm and are not palpable after menopause. The sacral ligaments relax, and the pelvic musculature weakens; consequently, the uterus droops. The cervix shrinks and looks paler with a thick glistening epithelium. The vaginal epithelium atrophies, becoming thinner, drier, and itchy. The vaginal pH becomes more alkaline, and secretions are decreased, which results in a fragile mucosal surface that is at risk for vaginitis.

During a vaginal examination of a 38-year-old woman, the nurse notices that the vulva and vagina are erythematous and edematous with thick, white, curdlike discharge adhering to the vaginal walls. The woman reports intense pruritus and thick white discharge from her vagina. The nurse knows that these history and physical examination findings are most consistent with which condition? a. Candidiasis b. Trichomoniasis c. Atrophic vaginitis d. Bacterial vaginosis

ANS: A The woman with candidiasis often reports intense pruritus and thick white discharge. The vulva and vagina are erythematous and edematous. The discharge is usually thick, white, and curdlike. Infection with trichomoniasis causes a profuse, watery, gray-green, and frothy discharge. Bacterial vaginosis causes a profuse discharge that has a "foul, fishy, rotten" odor. Atrophic vaginitis may have a mucoid discharge.

Of the 33 vertebrae in the spinal column, which is correct? a. 5 lumbar b. 5 thoracic c. 7 sacral d. 12 cervical

ANS: A There are 7 cervical, 12 thoracic, 5 lumbar, 5 sacral, and 3 to 4 coccygeal vertebrae in the spinal column.

Which of these tests would the nurse use to check the motor coordination of an 11-month-old infant? a. Denver II b. Stereognosis c. Deep tendon reflexes d. Rapid alternating movements

ANS: A To screen gross and fine motor coordination, the nurse should use the Denver II with its age-specific developmental milestones. Stereognosis tests a person's ability to recognize objects by feeling them and is not appropriate for an 11-month-old infant. Testing the deep tendon reflexes is not appropriate for checking motor coordination. Testing rapid alternating movements is not appropriate for testing coordination in infants or children. To screen gross and fine motor coordination, the nurse should use the Denver II with its age-specific developmental milestones.

A man who has had gout for several years comes to the clinic with a problem with his toe. On examination, the nurse notices the presence of hard, painless nodules over the great toe; one has burst open with a chalky discharge. What is this called? a. Tophi b. Callus c. Bunion d. Plantar wart

ANS: A Tophi are collections of monosodium urate crystals resulting from chronic gout in and around the joint that cause extreme swelling and joint deformity. They appear as hard, painless nodules (tophi) over the metatarsophalangeal joint of the first toe and they sometimes burst with a chalky discharge. A callus is a hard, thickened area of skin that forms as a result of friction or pressure. A bunion is a bony bump that forms on the joint at the base of your big toe (metatarsophalangeal joint). A plantar wart is vascular papillomatous growth that occurs on the sole of the foot, commonly at the ball and has small dark spots and is painful. A callus is a hard, thickened area of skin that forms as a result of friction or pressure. A bunion is a bony bump that forms on the joint at the base of your big toe (metatarsophalangeal joint). A plantar wart is vascular papillomatous growth that occurs on the sole of the foot, commonly at the ball and has small dark spots and is painful.

During the taking of the health history, a patient tells the nurse that "it feels like the room is spinning around me." How should the nurse document this finding? a. Vertigo b. Syncope c. Dizziness d. Seizure activity

ANS: A True vertigo is rotational spinning caused by a neurologic dysfunction or a problem in the vestibular apparatus or the vestibular nuclei in the brainstem. Syncope is a sudden loss of strength or a temporary loss of consciousness. Dizziness is a light-headed, swimming sensation. Seizure activity is characterized by altered or loss of consciousness, involuntary muscle movements, and sensory disturbances.

A 59-year-old patient has a herniated intervertebral disk. Which of the following findings should the nurse expect to see on physical assessment of this individual? a. Hyporeflexia b. Increased muscle tone c. Positive Babinski sign d. Presence of pathologic reflexes

ANS: A With a herniated intervertebral disk or lower motor neuron lesion, loss of tone, flaccidity, atrophy, fasciculations, and hyporeflexia or areflexia are demonstrated. No Babinski sign or pathologic reflexes would be observed. The other options reflect a lesion of upper motor neurons.

During a physical examination, the nurse finds that a male patient's foreskin is fixed and tight and will not retract over the glans. What is this condition called? a. Phimosis b. Epispadias c. Peyronie disease d. Urethral stricture

ANS: A With phimosis, the foreskin is nonretractable, forming a pointy tip of the penis with a tiny orifice at the end of the glans. The foreskin is advanced and so tight that it is impossible to retract over the glans. This condition may be congenital or acquired from adhesions related to infection. Epispadias is when the meatus opens on the dorsal (upper) side of glans or shaft above a broad, spadelike penis. It is rare but more disabling than hypospadias because of associated urinary incontinence and separation of pubic bones. A urethral stricture is a narrowing of the urethra which appears as a pinpoint, constricted opening at the meatus or inside along the urethra. A gradual decrease in force and caliber of urine stream is the most common symptom. Peyronie disease presents as subcutaneous plaques on the penis and is associated with painful bending of the penis during erection. Its cause is trauma to the penis with resulting scar, deformity, and often erectile dysfunction. The physical findings from this patient's examination indicate phimosis.

A 16-year-old boy is brought to the clinic for a problem that he refused to let his mother see. The nurse examines him, and finds that he has scrotal swelling on the left side. He had the mumps the previous week, and the nurse suspects that he has orchitis. Which of the following assessment findings support this diagnosis? (Select all that apply.) a. Swollen testis b. Mass that transilluminates c. Scrotal skin that is reddened d. Mass that does not transilluminate e. Scrotum that is tender upon palpation f. Scrotum that is nontender upon palpation

ANS: A, C, D, E With orchitis, the testis is swollen, with a feeling of weight, and is tender or painful. The mass does not transilluminate, and the scrotal skin is reddened. Transillumination of a mass occurs with a hydrocele, not orchitis.

A 69-year-old patient has been admitted to an adult psychiatric unit because his wife thinks he is getting more and more confused. He laughs when he is found to be forgetful, saying "I'm just getting old!" After the nurse completes a thorough neurologic assessment, which findings would be indicative of Alzheimer disease? (Select all that apply.) a. Getting lost in one's own neighborhood b. Occasionally forgetting names or appointments c. Sometimes having trouble finding the right word d. Misplacing items, such as putting dish soap in the refrigerator e. Difficulty performing familiar tasks, such as placing a telephone call f. Rapid mood swings, from calm to tears, for no apparent reason.

ANS: A, D, E, F Difficulty performing familiar tasks, misplacing items, rapid mood swings, and getting lost in one's own neighborhood can be warning signs of Alzheimer disease. Occasionally forgetting names or appointments, and sometimes having trouble finding the right word are part of normal aging

The nurse is assessing the breasts of a 68-year-old woman and discovers a mass in the upper outer quadrant of the left breast. When assessing this mass, the nurse is aware that characteristics of a cancerous mass include which of the following? (Select all that apply.) a. Nontender mass b. Regular border c. Hard, dense, and immobile d. Rubbery texture and mobile e. Dull, heavy pain on palpation f. Irregular, poorly delineated border

ANS: A, F Cancerous breast masses are solitary, unilateral, and nontender. They are solid, hard, dense, and fixed to underlying tissues or skin as cancer becomes invasive. Their borders are irregular and poorly delineated. They are often painless, although the person may experience pain. They are most common in the upper outer quadrant. A dull, heavy pain on palpation and a mass with a rubbery texture and a regular border are characteristics of benign breast disease.

In performing a breast examination, the nurse knows that examining the upper outer quadrant of the breast is especially important. What is the reason for this? a. It is the largest quadrant of the breast. b. It is the most common location of breast tumors. c. It is where the majority of suspensory ligaments attach. d. It is more prone to injury and calcifications than other locations in the breast.

ANS: B The upper outer quadrant is the site of most breast tumors. In the upper outer quadrant, the nurse should notice the axillary tail of Spence, the cone-shaped breast tissue that projects up into the axilla, close to the pectoral group of axillary lymph nodes. The other options are incorrect.

When performing a genital examination on a 25-year-old man, the nurse notices deeply pigmented, wrinkled scrotal skin with large sebaceous follicles. On the basis of this information, how should the nurse proceed? a. Squeeze the glans to check for the presence of discharge. b. Consider this finding as normal, and proceed with the examination. c. Assess the testicles for the presence of masses or painless lumps. d. Obtain a more detailed history, focusing on any scrotal abnormalities the patient has noticed.

ANS: B After adolescence, the scrotal skin is deeply pigmented and has large sebaceous follicles and appears corrugated.

During a breast health interview, a patient states that she has noticed pain in her left breast. Which statement by the nurse is most appropriate? a. "Don't worry about the pain; breast cancer is not painful." b. "I would like some more information about the pain in your left breast." c. "Oh, I had pain like that after my son was born; it turned out to be a blocked milk duct." d. "Breast pain is almost always the result of benign breast disease."

ANS: B Although breast cancer usually occurs from trauma, inflammation, infection, or benign breast disease, rather than cancer, the nurse should gather more information about the patient's pain. The nurse should not belittle the patient's feelings by using statements like "don't worry" or by sharing personal experiences.

If an imaginary line were drawn connecting the highest point on each iliac crest. What vertebra would that line cross? a. First sacral b. Fourth lumbar c. Seventh cervical d. Twelfth thoracic

ANS: B An imaginary line connecting the highest point on each iliac crest crosses the fourth lumbar vertebra. The other options are not correct.

The assessment of a 60-year-old patient has taken longer than anticipated. In testing his pain perception, the nurse decides to complete the test as quickly as possible. When the nurse applies the sharp point of the pin on his arm several times, he is only able to identify these as one "very sharp prick." What would be the most accurate explanation for this? a. The patient has hyperesthesia as a result of the aging process. b. This response is most likely the result of the summation effect. c. The nurse was probably not poking hard enough with the pin in the other areas. d. The patient most likely has analgesia in some areas of arm and hyperalgesia in others.

ANS: B At least 2 seconds should be allowed to elapse between each stimulus to avoid summation. With summation, frequent consecutive stimuli are perceived as one strong stimulus. The other responses are incorrect.

A 32-year-old woman tells the nurse that she has noticed "very sudden, jerky movements" mainly in her hands and arms. She says, "They seem to come and go, primarily when I am trying to do something. I haven't noticed them when I'm sleeping." What do these symptoms suggest? a. Tics b. Chorea c. Athetosis d. Myoclonus

ANS: B Chorea is characterized by sudden, rapid, jerky, purposeless movements that involve the limbs, trunk, or face. Chorea occurs at irregular intervals, and the movements are all accentuated by voluntary actions. A tic is an involuntary, compulsive, repetitive twitching of a muscle group (e.g. wink, grimace, head movement, shoulder shrug); due to a neurologic cause or a psychogenic cause. Athetosis is slow, twisting, writhing, continuous movement, resembling a snake or worm. Myoclonus is rapid, sudden jerk or a short series of jerks at fairly regular intervals (a hiccup is a myoclonus of the diaphragm). .

What controls humans' ability to perform very skilled movements such as writing? a. Basal ganglia b. Corticospinal tract c. Spinothalamic tract d. Extrapyramidal tract

ANS: B Corticospinal fibers mediate voluntary movement, particularly very skilled, discrete, and purposeful movements, such as writing. The corticospinal tract, also known as the pyramidal tract, is a newer, "higher" motor system that humans have that permits very skilled and purposeful movements. The other responses are not r/t skilled movements. The basal ganglia are large bands of gray matter buried deep within the two cerebral hemispheres that from the subcortical-associated motor system and help to initiate and coordinate movement and control automatic associated movements of the body (e.g. arm swing alternating with the legs during walking). The spinothalamic tract is one of the major sensory pathways of the CNS and has two parts. The lateral spinothalamic tract carries pain and temperature sensations and the anterior spinothalamic tract carries crude touch. The extrapyramidal tracts include all the motor nerve fibers originating in the motor cortex, basal ganglia, brainstem, and spinal cord that are outside the pyramidal tract and maintains muscle tone and control body movements, especially gross automatic movements such as walking.

A 54-year-old woman who has just completed menopause is in the clinic today for a yearly physical examination. Which of these statements should the nurse include in patient education? a. "You can continue with hormone replacement therapy as it actually decreases your risk for breast cancer." b. "You should be aware that you're at increased risk for dyspareunia because of decreased vaginal secretions." c. "You have only stopped menstruating and there are not really any other changes that you need to be concerned about." d. "You likely may have difficulty with sexual pleasure as a result of drastic changes in the female sexual response cycle."

ANS: B Decreased vaginal secretions leave the vagina dry and at risk for irritation and pain with intercourse (dyspareunia). Hormone replacement therapy increases, not decreases, the risk for breast cancer. In addition to cessation of menses, there are several other changes that occur with menopause. The female's hormonal milieu decreases rapidly, the uterus shrinks, the ovaries atrophy, the pelvic musculature weakens, the cervix shrinks, and the vagina becomes shorter, narrower, less elastic, and vaginal epithelium atrophies, becoming thinner, drier, and itchy. However, these physical changes need not affect sexual pleasure and function.

During an examination, the nurse notices severe nystagmus in both eyes of a patient. Which conclusion by the nurse is correct? a. A normal occurrence b. Indicates disease of the cerebellum or brainstem c. A sign that the patient is nervous about the examination d. Indicates a visual problem, and a referral to an ophthalmologist is indicated

ANS: B End-point nystagmus at an extreme lateral gaze normally occurs; however, the nurse should carefully assess any other nystagmuses. Severe nystagmus occurs with disease of the vestibular system, cerebellum, or brainstem

A 55-year-old man is experiencing severe pain of sudden onset in the scrotal area. It is somewhat relieved by elevation. On examination the nurse notices an enlarged, red scrotum that is very tender to palpation. Distinguishing the epididymis from the testis is difficult, and the scrotal skin is thick and edematous. What do these findings suggest? a. A varicocele b. Epididymitis c. A spermatocele d. Testicular torsion

ANS: B Epididymitis presents as severe pain of sudden onset in the scrotum that is somewhat relieved by elevation. On examination, the scrotum is enlarged, reddened, and exquisitely tender. The epididymis is enlarged and indurated and may be hard to distinguish from the testis. The overlying scrotal skin may be thick and edematous. A varicocele can present with either a dull pain, constant pulling or dragging sensation, or be asymptomatic. Appearance upon inspection may be normal or the lighter scrotal skin may have a bluish color and the testis on side of varicocele may be smaller due to impaired circulation. When standing a soft irregular mass posterior to and above testis which feels like a "bag of worms" may be palpable and collapses when supine and then refills when upright. A spermatocele is usually a painless, round, freely movable mass lying above and behind testis and if large may feel like a third testis. Testicular torsion presents with sudden onset of excruciating unilateral pain in testicle; red, swollen scrotum with one testes (usually the left) higher owing to rotation of shortening; and extremely tender to palpation and difficult to distinguish epididymis from testis.

A woman who has had rheumatoid arthritis for years is starting to notice that her fingers are drifting to the side. What is term commonly used for this condition? a. Radial drift b. Ulnar deviation c. Swan-neck deformity d. Dupuytren contracture

ANS: B Fingers drift to the ulnar side because of stretching of the articular capsule and muscle imbalance caused by chronic rheumatoid arthritis. A radial drift is not observed. Swan-neck deformity is a flexion contracture in the metacarpophalangeal joint, then hyperextension of the PIP joint, and flexion of the DIP joint which resembles the curve of a swan's neck. Dupuytren contracture is a flexion contracture of the digits. It first affects the fourth digit, then the fifth digit, and then third digit.

The nurse is preparing to examine the external genitalia of a school-age girl. Which position would be most appropriate in this situation? a. In the parent's lap b. In a frog-leg position on the examining table c. In the lithotomy position with the feet in stirrups d. Lying flat on the examining table with legs extended

ANS: B For school-age children, placing them on the examining table in a frog-leg position is best. With toddlers and preschoolers, having the child on the parent's lap in a frog-leg position is best.

During a genital examination, the nurse notices that a male patient has clusters of small vesicles on the glans, surrounded by erythema. What does this finding suggest? a. Genital warts b. Genital herpes c. Peyronie disease d. Syphilitic chancres

ANS: B Genital herpes, or herpes simplex virus 2 (HSV-2), infections present as clusters of small vesicles with surrounding erythema, which are often painful and erupt on the glans or foreskin. The lesions of genital warts are soft, pointed, moist, fleshy, painless papules that may be single or multiple in a cauliflower-like patch. They occur on the shaft of the penis, behind the corona, or around the anus, where they may grow into large grapelike clusters. Peyronie disease presents as subcutaneous plaques on the penis and is associated with painful bending of the penis during erection. Its cause is trauma to the penis with resulting scar, deformity, and often erectile dysfunction. Syphilitic chancres begin within 2 to 4 weeks of infection as a small, solitary, silvery papule that erodes to a red, round or oval, superficial ulcer with a yellowish serous discharge. The symptoms this patient is presenting with are those of genital herpes.

A woman has just been diagnosed with HPV or genital warts. The nurse should counsel her to receive regular examinations because this virus makes her at a higher risk for what type of cancer? a. Uterine b. Cervical c. Ovarian d. Endometrial

ANS: B HPV is the virus responsible for most cases of cervical cancer, not the other options.

During an external genitalia examination of a woman, the nurse notices several lesions around the vulva. The lesions are pink, moist, soft, and pointed papules. The patient states that she is not aware of any problems in that area. What do these findings likely indicate? a. Syphilitic chancre b. HPV or genital warts c. Pediculosis pubis (crab lice) d. Herpes simplex virus type 2 (herpes genitalis)

ANS: B HPV lesions are painless, warty growths that the woman may not notice. Lesions are pink or flesh colored, soft, pointed, moist, warty papules that occur in single or multiple cauliflower-like patches around the vulva, introitus, anus, vagina, or cervix. Herpetic lesions are painful clusters of small, shallow vesicles with surrounding erythema. Syphilitic chancres begin as a solitary silvery papule that erodes into a red, round or oval superficial ulcer with a yellowish discharge. Pediculosis pubis causes severe perineal itching and excoriations and erythematous areas.

A 30-year-old woman is visiting the clinic because of "pain in my bottom when I have a bowel movement." The nurse should assess for which problem? a. Pinworms b. Hemorrhoids c. Colon cancer d. Fecal incontinence

ANS: B Having painful bowel movements, known as dyschezia, may be attributable to a local condition (hemorrhoid or fissure) or constipation. Pinworms cause acute pain, itching, and a papular rash. Colon cancer often presents with occult blood in the stool. Fecal incontinence is leaking of solid or liquid stool involuntarily.

A patient states during the interview that she noticed a new lump in the shower a few days ago. It was on her left breast near her axilla. How should the nurse proceed? a. Palpate the lump first. b. Palpate the unaffected breast first. c. Avoid palpating the lump because it could be a cyst, which might rupture. d. Palpate the breast with the lump first but plan to palpate the axilla last.

ANS: B If the woman mentions a breast lump she has discovered herself, then the nurse should examine the unaffected breast first to learn a baseline of normal consistency for this individual.

An older man is concerned about his sexual performance. In addition to a disease, what else should the nurse explain can cause a withdrawal from sexual activity later in life? a. Decreased sperm production b. Side effects of medications c. Decreased libido with aging d. Decreased pleasure from sexual intercourse

ANS: B In the absence of disease, a withdrawal from sexual activity may be attributable to side effects of medications such as antihypertensives, antidepressants, sedatives, psychotropics, antispasmotics, tranquilizers or narcotics, and estrogens; loss of spouse; depression; preoccupation with work; marital or family conflict; heavy use of alcohol; lack of privacy (living with adult children or in nursing home); economic or emotional stress; poor nutrition; or fatigue. Although there is a decrease in sperm production and other physical changes with aging, they need not interfere with the libido and pleasure from sexual intercourse

A married couple has come to the clinic seeking advice on pregnancy. They have been trying to conceive for 4 months and have not been successful. What should the nurse do first? a. Ascertain whether either of them has been using broad-spectrum antibiotics. b. Explain that couples are considered infertile after 1 year of unprotected intercourse. c. Immediately refer the woman to an expert in pelvic inflammatory disease—the most common cause of infertility. d. Explain that couples are considered infertile after 3 months of engaging in unprotected intercourse and that they will need a referral to a fertility expert.

ANS: B Infertility is considered after 1 year of engaging in unprotected sexual intercourse without conceiving. The other actions are not appropriate.

What are the functional units of the musculoskeletal system? a. Bones b. Joints c. Muscles d. Tendons

ANS: B Joints are the functional units of the musculoskeletal system because they permit the mobility needed to perform the activities of daily living. The skeleton (bones) is the framework of the body. There are three types of muscles: skeletal, smooth, and cardiac and they produce movement when they contract. Tendons are strong fibrous cords that attach skeletal muscles to the bones. The other options are not correct.

What does testing kinesthesia assess? a. Fine touch b. Position sense c. Motor coordination d. Perception of vibration

ANS: B Kinesthesia, or position sense, is the person's ability to perceive passive movements of the extremities. Fine touch is assessed by the stereognosis, graphesthesia, extinction, and point location tests. Motor coordination is assessed by the Denver II test and reflexes. Perception of vibration is assessed by hitting a tuning fork so that it is vibrating and placing it on boney prominences

An 80-year-old woman is visiting the clinic for a checkup. She states, "I can't walk as much as I used to." What should the nurse have the patient do to observe for motor dysfunction in her hip? a. Internally rotate her hip while she is sitting. b. Abduct her hip while she is lying on her back. c. Adduct her hip while she is lying on her back. d. Externally rotate her hip while she is standing

ANS: B Limited abduction of the hip while supine is the most common motion dysfunction found in hip disease. The other options are not correct.

The nurse has just completed an inspection of a nulliparous woman's external genitalia. Which of these would be a description of a finding within normal limits? a. Redness of the labia majora b. Multiple nontender sebaceous cysts c. Gaping and slightly shriveled labia majora d. Discharge that is foul smelling and irritating

ANS: B No lesions should be noted, except for the occasional sebaceous cysts, which are yellowish 1-cm nodules that are firm, nontender, and often multiple. The labia majora are dark pink, moist, and symmetric; redness indicates inflammation or lesions. Discharge that is foul smelling and irritating may indicate infection. In the nulliparous woman, the labia majora meet in the midline, are symmetric and plump.

While inspecting a patient's breasts, the nurse finds that the left breast is slightly larger than the right with the bilateral presence of Montgomery glands. How should the nurse proceed? a. Palpate over the Montgomery glands, checking for drainage. b. Consider these findings as normal, and proceed with the examination. c. Ask extensive health history questions regarding the woman's breast asymmetry. d. Continue with the examination, and then refer the patient for further evaluation of the Montgomery glands.

ANS: B Normal findings of the breast include one breast (most often the left) slightly larger than the other and the presence of Montgomery glands across the areola. These are normal findings so there is no need to palpate the Montgomery glands and checking for drainage, asking extensive health history questions, or referring the patient for further evaluation of the Montgomery glands

When performing a scrotal assessment, the nurse notices that the scrotal contents show a red glow with transillumination. How should the nurse proceed? a. Assess the patient for the presence of a hernia. b. Suspect the presence of serous fluid in the scrotum. c. Refer the patient for evaluation of a mass in the scrotum. d. Consider this finding normal and proceed with the examination.

ANS: B Normal scrotal contents do not allow light to pass through the scrotum. However, serous fluid does transilluminate and shows as a red glow. Neither a mass nor a hernia would transilluminate.

The nurse is palpating a female patient's adnexa. The findings include a firm, smooth uterine wall; the ovaries are palpable and feel smooth and firm. The fallopian tube is firm and pulsating. How should the nurse proceed? a. Tell the patient that her examination is normal. b. Give her an immediate referral to a gynecologist. c. Suggest that she return in a month for a recheck to verify the findings. d. Tell the patient that she may have an ovarian cyst that should be evaluated further.

ANS: B Normally the uterine wall feels firm and smooth, with the contour of the fundus rounded. Ovaries are not often palpable, but when they are, they normally feel smooth, firm, and almond shaped and are highly movable, sliding through the fingers. The fallopian tube is not normally palpable. No other mass or pulsation should be felt. Pulsation or palpable fallopian tube suggests ectopic pregnancy, which warrants immediate referral.

During the health history of a patient who reports chronic constipation, the patient asks the nurse about foods to eat to avoid constipation. What should the nurse include as an example of an appropriate food? a. Yogurt b. Broccoli c. Ground beef d. Iceberg lettuce

ANS: B Patients with constipation should be encouraged to eat foods high in fiber. High-fiber foods are either soluble type (e.g., beans, prunes, barley, broccoli) or insoluble type (e.g., cereals, wheat germ). The other examples are not considered high-fiber foods

The mother of a 10-year-old boy asks the nurse about the recognition of puberty. How should the nurse reply? a. "Puberty usually begins around 15 years of age." b. "The first sign of puberty is an enlargement of the testes." c. "The penis size does not increase until about 16 years of age." d. "The development of pubic hair precedes testicular or penis enlargement."

ANS: B Puberty begins sometime between age 9 for African Americans and age 10 for Caucasians and Hispanics. The first sign is an enlargement of the testes. Pubic hair appears next, and then penis size increases.

When assessing the scrotum of a male patient, the nurse notices the presence of multiple firm, nontender, yellow 1-cm nodules. What does this finding indicate? a. Urethritis b. Sebaceous cysts c. Subcutaneous plaques d. Due to an inflammation of the epididymis

ANS: B Sebaceous cysts are commonly found on the scrotum. These yellowish 1-cm nodules are firm, nontender, and often multiple. Urethritis is infection of the urethra which causes painful, burning urination or pruritis. Meatus edges are reddened, everted, and swollen with purulent drainage. Subcutaneous plaque on the penis is called Peyronie disease and is associated with painful bending of the penis during erection. Its cause is trauma to the penis with resulting scar, deformity, and often erectile dysfunction. Inflammation of the epididymis (epididymitis) causes swelling and severe pain of sudden onset in the scrotum, which is relieved by elevation. The multiple yellowish 1 cm nodules this patient has are sebaceous cysts. Sebaceous cysts are commonly found on the scrotum. These yellowish 1-cm nodules are firm, nontender, and often multiple.

A 50-year-old woman calls the clinic because she has noticed some changes in her body and breasts and wonders if these changes could be attributable to the hormone replacement therapy (HRT) she started 3 months earlier. How should the nurse respond? a. "HRT is at such a low dose that side effects are very unusual." b. "HRT has several side effects, including fluid retention, breast tenderness, and vaginal bleeding." c. "Vaginal bleeding with HRT is very unusual; I suggest you come into the clinic immediately to have this evaluated." d. "It sounds as if your dose of estrogen is too high; I think you may need to decrease the amount you are taking and then call back in a week."

ANS: B Side effects of HRT include fluid retention, breast pain, and vaginal bleeding. The other responses are not correct.

The nurse places a key in the hand of a patient and he identifies it as a penny. What term would the nurse use to describe this finding? a. Extinction b. Stereognosis c. Graphesthesia d. Tactile discrimination

ANS: B Stereognosis is the person's ability to recognize objects by feeling their forms, sizes, and weights. Astereognosis is an inability to identify objects correctly, and it occurs in sensory cortex lesions. Extinction tests the person's ability to feel sensations on both sides of the body at the same point. Graphesthesia is the ability to "read" a number by having it traced on the skin. Tactile discrimination tests fine touch.

The nurse is performing an examination of the anus and rectum. Which of these statements is correct and important to remember during this examination? a. The rectum is approximately 8 cm long. b. The anorectal junction cannot be palpated. c. Above the anal canal, the rectum turns anteriorly. d. There are no sensory nerves in the anal canal or rectum.

ANS: B The anal columns are folds of mucosa that extend vertically down from the rectum and end in the anorectal junction. This junction is not palpable but is visible on proctoscopy. The rectum is 12 cm long; just above the anal canal, the rectum dilates and turns posteriorly.

if a patient reports a recent breast infection, then the nurse should expect to find what type of node enlargement? a. Nonspecific b. Ipsilateral axillary c. Inguinal and cervical d. Contralateral axillary

ANS: B The breast has extensive lymphatic drainage. Most of the lymph, more than 75%, drains into the ipsilateral, or same side, axillary nodes. If there was a recent breast infection, then the same side (ipsilateral) axillary nodes will likely be enlarged. A recent breast infection would not cause enlargement of the inguinal or cervical lymph nodes or lymph nodes on the opposite (contralateral) side. Instead, since the breast has extensive lymphatic drainage with most of the lymph (more than 75%) drains into the ipsilateral, or same side, axillary nodes. If there was a recent breast infection, then the same side (ipsilateral) axillary nodes will likely be enlarged.

During an interview the patient states, "I can feel this bump on the top of both of my shoulders—it doesn't hurt but I am curious about what it might be." What should the nurse tell this patient? a. "That is the subacromial bursa." b. "That is the acromion process." c. "That is the glenohumeral joint." d. "That is the greater tubercle of the humerus."

ANS: B The bump of the scapula's acromion process is felt at the very top of the shoulder. The other options are not correct

During an internal examination of a woman's genitalia, the nurse will use which technique for proper insertion of the speculum? a. The woman is instructed to bear down, the speculum blades are opened and applied in a swift, upward movement. b. The woman is instructed to bear down, the width of the blades is horizontally turned, and the speculum is inserted downward at a 45-degree angle toward the small of the woman's back. c. The blades of the speculum are inserted on a horizontal plane, turning them to a 30-degree angle while continuing to insert them. The woman is asked to bear down after the speculum is inserted. d. The blades are locked open by turning the thumbscrew. Once the blades are open, pressure is applied to the introitus and the blades are inserted downward at a 45-degree angle to bring the cervix into view.

ANS: B The examiner should instruct the woman to bear down, turn the width of the blades horizontally, and insert the speculum at a 45-degree angle downward toward the small of the woman's back.

When the nurse is discussing sexuality and sexual issues with an adolescent, a permission statement helps convey that it is normal to think or feel a certain way. Which statement is the best example of a permission statement? a. "It is okay that you have become sexually active." b. "Girls your age often have questions about sexual activity. Do you have any questions?" c. "If it is okay with you, I'd like to ask you some questions about your sexual history." d. "Girls your age often engage in sexual activities. It is okay to tell me if you have had intercourse."

ANS: B The examiner should start with a permission statement such as, "Girls your age often experience..." A permission statement conveys the idea that it is normal to think or feel a certain way, and implying that the topic is normal and unexceptional is important.

START OF MALE GENITOURINARY SYSTEM QUESTIONS: Which is a structure of the external male genital? a. Testis b. Scrotum c. Epididymis d. Vas deferens

ANS: B The external male genital structures include the penis and scrotum. The testis, epididymis, and vas deferens are internal structures.

Which statement concerning the sphincters is correct? a. The internal sphincter is under voluntary control. b. The external sphincter is under voluntary control. c. Both sphincters remain slightly relaxed at all times. d. The internal sphincter surrounds the external sphincter

ANS: B The external sphincter not only surrounds the internal sphincter but also has a small section overriding the tip of the internal sphincter at the opening. The external sphincter is under voluntary control. Except for the passing of feces and gas, the sphincters keep the anal canal tightly closed.

A 55-year-old postmenopausal woman is being seen in the clinic for her annual examination. She is concerned about changes in her breasts that she has noticed over the past 5 years. She states that her breasts have decreased in size and that the elasticity has changed so that her breasts seem "flat and flabby." Which is the best reply by the nurse? a. "This change occurs most often because of long-term use of bras that do not provide enough support to the breast tissues." b. "Decreases in hormones after menopause causes atrophy of the glandular tissue in the breast and is a normal process of aging." c. "This is a normal change that occurs as women get older and is due to the increased levels of progesterone during the aging process." d. "Postural changes in the spine make it appear that your breasts have changed in shape. Exercises to strengthen the muscles of the upper back and chest wall will help prevent the changes in elasticity and size."

ANS: B The hormonal changes of menopause cause the breast glandular tissue to atrophy, making the breasts more pendulous, flattened, and sagging. This change in the breasts is not due to long-term use of unsupportive bras, increased levels of progesterone with aging, or postural changes in the spine. Rather, the hormonal changes of menopause cause the breast glandular tissue to atrophy, making the breasts more pendulous, flattened, and sagging.

Which statement concerning the areas of the brain is true? a. The cerebellum is the center for speech and emotions. b. The hypothalamus controls body temperature and regulates sleep. c. The basal ganglia are responsible for controlling voluntary movements. d. Motor pathways of the spinal cord and brainstem synapse in the thalamus.

ANS: B The hypothalamus is a vital area with many important functions: body temperature controller, sleep center, anterior and posterior pituitary gland regulator, and coordinator of autonomic nervous system activity and emotional status. The cerebellum controls motor coordination, equilibrium, and balance. The basal ganglia control autonomic movements of the body. The motor pathways of the spinal cord synapse in various areas of the spinal cord, not in the thalamus.

The nurse is preparing for a class on breast cancer. Which statement is true with regard to cultural differences in breast cancer in the United States? a. Black women have a lower incidence of aggressive, triple negative breast cancer. b. The relative 5-year survival rate for black women is lower than that for Caucasian women. c. For every stage of breast cancer, Asian/Pacific Islander women have the lowest rate of survival. d. Ashkenazi Jewish women have a significantly lower prevalence of BRCA1 and BRCA2 gene mutations.

ANS: B The incidence of breast cancer and survival rates varies between different cultural groups. The relative 5-year survival rate is 83% for black women and 92% for white women. Survival varies by the stage of cancer when it is diagnosed and black women are diagnosed at a later stage and have higher rates of the aggressive, triple negative breast cancer. The incidence of breast cancer and survival rates varies between different cultural groups. The relative 5-year survival rate for black women is lower (not higher) than Caucasian women (83% to 92%). Survival varies by the stage of cancer when it is diagnosed and black women are diagnosed at a later stage and have higher rates (not lower incidence) of the aggressive, triple negative breast cancer. For every stage of breast cancer, Asian/Pacific Islander women have the highest (not lowest) rate of survival and Ashkenazi Jewish women have a significantly higher (not lower) prevalence of BRCA1 and BRCA2 gene mutations.

A mother of a 1-month-old infant asks the nurse why it takes so long for infants to learn to roll over. What is the reason for this? a. A demyelinating process must be occurring with her infant. b. Myelin is needed to conduct the impulses, and the neurons of a newborn are not yet myelinated. c. The cerebral cortex is not fully developed; therefore, control over motor function gradually occurs. d. The spinal cord is controlling the movement because the cerebellum is not yet fully developed.

ANS: B The infant's sensory and motor development proceeds along with the gradual acquisition of myelin, which is needed to conduct most impulses. Very little cortical control exists, and the neurons are not yet myelinated. The other responses are not correct.

The nurse is assessing a patient's ischial tuberosity. How should the nurse position the patient to palpate the ischial tuberosity? a. Standing b. Flexing the hip c. Flexing the knee d. Lying in the supine position

ANS: B The ischial tuberosity lies under the gluteus maximus muscle and is palpable when the hip is flexed. The other options are not correct.

The nurse is checking the range of motion in a patient's knee and knows that the knee is capable of which movement(s)? a. Circumduction b. Flexion and extension c. Inversion and eversion d. Supination and pronation

ANS: B The knee is a hinge joint, permitting flexion and extension of the lower leg on a single plane. The knee is not capable of circumduction, inversion, eversion, supination, or pronation.

When performing a genital assessment on a middle-aged man, the nurse notices multiple soft, moist, painless papules in the shape of cauliflower-like patches scattered across the shaft of the penis. What does this finding suggest? a. Carcinoma b. Genital warts c. Genital herpes d. Syphilitic chancres

ANS: B The lesions of genital warts are soft, pointed, moist, fleshy, painless papules that may be single or multiple in a cauliflower-like patch. They occur on the shaft of the penis, behind the corona, or around the anus, where they may grow into large grapelike clusters. Genital carcinoma begins as red, raised, warty growth or as an ulcer with watery discharge which almost always occur on the glans or inner lip of foreskin. Genital herpes (HSV-2 infection) appears as clusters of small vesicles with surrounding erythema which are often painful and erupt on the glans, foreskin, or anus. Syphilitic chancres begin within 2 to 4 weeks of infection as a small, solitary, silvery papule that erodes to a red, round or oval, superficial ulcer with a yellowish serous discharge. The symptoms this patient is experiencing are those of genital warts.

START OF NEUROLOGIC SYSTEM QUESTIONS: What are the two parts of the nervous system? a. Motor and sensory b. Central and peripheral c. Peripheral and autonomic d. Hypothalamus and cerebral

ANS: B The nervous system can be divided into two parts—central and peripheral. The central nervous system includes the brain and spinal cord. The peripheral nervous system includes the 12 pairs of cranial nerves (CNs), the 31 pairs of spinal nerves, and all of their branches. Motor and sensory refer to the two types of nerve tract pathways in the CNS. Peripheral and autonomic both are part of the peripheral part of the nervous system. The peripheral nervous system has two parts, the somatic and autonomic. The hypothalamus and cerebral are parts of the brain.

During the examination portion of a patient's visit, she will be in lithotomy position. Which statement reflects some things that the nurse can do to make this position more comfortable for her? a. Ask her to place her hands and arms over her head. b. Elevate her head and shoulders to maintain eye contact. c. Allow her to choose to have her feet in the stirrups or have them resting side by side on the edge of the table. d. Allow her to keep her buttocks approximately 6 inches from the edge of the table to prevent her from feeling as if she will fall off.

ANS: B The nurse should elevate her head and shoulders to maintain eye contact. The patient's arms should be placed at her sides or across the chest. Placing her hands and arms over her head only tightens the abdominal muscles. The feet should be placed into the stirrups, knees apart, and buttocks at the edge of the examining table. The stirrups are placed so that the legs are not abducted too far.

During an examination of a woman, the nurse notices that her left breast is slightly larger than her right breast. Which of these statements is true about this finding? a. Asymmetry of the breasts is unusual and the patient should be referred to physician. b. Asymmetry of the breasts is common, but the nurse should verify that this finding is not new. c. Asymmetry of breast size and shape is very unusual and means she may have an inflammation or growth. d. Asymmetry of breast size and shape is probably due to breastfeeding and is nothing to worry about.

ANS: B The nurse should notice symmetry of size and shape of the breasts. It is common to have a slight asymmetry in size; often the left breast is slightly larger than the right. However, a sudden increase in the size of one breast signifies inflammation or new growth. The nurse should verify that this asymmetry is not new.

During an assessment of the cranial nerves (CNs), the nurse finds the following: asymmetry when the patient smiles or frowns, uneven lifting of the eyebrows, sagging of the lower eyelids, and escape of air when the nurse presses against the right puffed cheek. These findings indicate dysfunction of which cranial nerve(s)? a. Motor component of CN IV b. Motor component of CN VII c. Motor and sensory components of CN XI d. Motor component of CN X and sensory component of CN VII

ANS: B The nurse's findings all reflect motor dysfunction, none are sensory. The specific cranial nerve affected is the facial nerve (CN VII). Cranial nerve IV, the trochlear nerve, innervates a muscle in the eye muscle and is responsible for eye movement, not the symptoms this patient is experiencing. The nurse's findings all reflect motor dysfunction, none are sensory, therefore options c and d can be eliminated because they each contain a sensory component.

During the neurologic assessment of a "healthy" 35-year-old patient, the nurse asks him to relax his muscles completely. The nurse then moves each extremity through full range of motion. Which of these results would the nurse expect to find? a. Firm, rigid resistance to movement b. Mild, even resistance to movement c. Slight pain with some directions of movement d. Hypotonic muscles as a result of total relaxation

ANS: B Tone is the normal degree of tension (contraction) in voluntarily relaxed muscles. It shows a mild resistance to passive stretching. Normally the nurse will notice a mild, even resistance to movement. The other responses are not correct.

During an examination of an aging man, what finding would the nurse expect? a. Change in scrotal color b. Decrease in the size of the penis c. Enlargement of the testes and scrotum d. Increase in the number of rugae over the scrotal sac

ANS: B When assessing the genitals of an older man, the nurse may notice thinner, graying pubic hair and a decrease in the size of the penis. The size of the testes may be decreased, they may feel less firm, and the scrotal sac is pendulous with less rugae. No change in scrotal color is observed.

A patient calls the clinic for instructions before having a Papanicolaou (Pap) smear. What is an appropriate response by the nurse? a. "If you are menstruating, please use pads to avoid placing anything into the vagina." b. "Avoid intercourse, inserting anything into the vagina, or douching within 24 hours of your appointment." c. "We would like you to use a mild saline douche before your examination. You may pick this up in our office." d. "If you suspect that you have a vaginal infection, please gather a sample of the discharge to bring with you."

ANS: B When instructing a patient before Pap smear is obtained, the nurse should follow these guidelines: Do not obtain during the woman's menses or if a heavy infectious discharge is present. Instruct the woman not to douche, have intercourse, or put anything into the vagina within 24 hours before collecting the specimens. Any specimens will be obtained during the visit, not beforehand

The nurse is preparing to interview a postmenopausal woman. Which of these statements is true as it applies to obtaining the health history of a postmenopausal woman? a. The nurse should screen for monthly breast tenderness. b. The nurse should ask a postmenopausal woman if she has ever had vaginal bleeding. c. Once a woman reaches menopause, the nurse does not need to ask any history questions. d. Postmenopausal women are not at risk for contracting STIs; therefore, these questions can be omitted.

ANS: B Postmenopausal bleeding warrants further workup and referral. The other statements are not true

The nurse is assessing the joints of a woman who has stated, "I have a long family history of arthritis, and my joints hurt." The nurse suspects that she has osteoarthritis. Which of these are symptoms of osteoarthritis? (Select all that apply.) a. Symmetric joint involvement b. Asymmetric joint involvement c. Pain with motion of affected joints d. Affected joints may have heat, redness, and swelling e. Affected joints are swollen with hard, bony protuberances

ANS: B, C, E In osteoarthritis, asymmetric joint involvement commonly affects hands, knees, hips, and lumbar and cervical segments of the spine. Affected joints have stiffness, swelling with hard bony protuberances, pain with motion, and limitation of motion. The other options reflect the signs of rheumatoid arthritis.

The nurse is examining a 62-year-old man and notes that he has bilateral gynecomastia. The nurse should explore his health history for which related conditions? (Select all that apply.) a. Malnutrition b. Liver disease c. Hyperthyroidism d. Type 2 diabetes mellitus e. History of alcohol abuse

ANS: B, C, E Gynecomastia occurs with Cushing syndrome, liver cirrhosis, adrenal disease, hyperthyroidism, and numerous drugs, such as alcohol and marijuana use, estrogen treatment for prostate cancer, antibiotics (metronidazole, isoniazid), digoxin, angiotensin-converting enzyme (ACE) inhibitors, diazepam, and tricyclic antidepressants.

While the nurse is taking the history of a 68-year-old patient who sustained a head injury 3 days earlier, he tells the nurse that he is on a cruise ship and is 30 years old. What does this finding indicate? a. Great sense of humor b. Uncooperative behavior c. Decreased level of consciousness d. Inability to understand questions

ANS: C A change in consciousness may be subtle. The nurse should notice any decreasing level of consciousness, disorientation, memory loss, uncooperative behavior, or even complacency in a previously combative person. The other responses are incorrect.

A patient has a severed spinal nerve as a result of trauma. Which statement is true in this situation? a. Because there are 31 pairs of spinal nerves, no effect results if only one nerve is severed. b. The dermatome served by this nerve will no longer experience any sensation. c. The adjacent spinal nerves will continue to carry sensations for the dermatome served by the severed nerve. d. A severed spinal nerve will only affect motor function of the patient because spinal nerves have no sensory component.

ANS: C A dermatome is a circumscribed skin area that is primarily supplied from one spinal cord segment through a particular spinal nerve. The dermatomes overlap, which is a form of biologic insurance; that is, if one nerve is severed, then most of the sensations can be transmitted by the spinal nerve above and the spinal nerve below the severed nerve.

During a digital examination of the rectum, the nurse notices that the patient has hard feces in the rectum. The patient complains of feeling "full," has a distended abdomen, and states that she has not had a bowel movement "for several days." The nurse suspects which condition? a. Rectal polyp b. Rectal abscess c. Fecal impaction d. Fecal incontinence

ANS: C A fecal impaction is a collection of hard, desiccated feces in the rectum. The obstruction often results from decreased bowel motility, in which more water is reabsorbed from the stool. Rectal polyps are protruding growths from the rectal mucous membrane. They are usually asymptomatic and are fairly common. A rectal abscess starts from an infected anorectal gland and the infection channels through the perianal tissues to form a fistula (connection between the infected gland and the outside perineum). Symptoms include persistent pain and swelling and may drain purulent or serosanguineous matter. Fecal incontinence is the leaking of solid or liquid stool involuntarily.

A teenage girl has arrived reporting pain in her left wrist. She was playing basketball when she fell and landed on her left hand. The nurse examines her hand. Which finding would lead the nurse to expect a fracture? a. Dull ache b. Deep pain in her wrist c. Sharp pain that increases with movement d. Dull throbbing pain that increases with rest

ANS: C A fracture causes sharp pain that increases with movement. The other types of pain do not occur with a fracture

The nurse is providing patient education for a man who has been diagnosed with a rotator cuff injury. When explaining the structures involved in his injury, what should the nurse include? a. Nucleus pulposus b. Medial epicondyle c. Glenohumeral joint d. Articular processes

ANS: C A rotator cuff injury involves the glenohumeral joint, which is enclosed by a group of four powerful muscles and tendons that support and stabilize it. The other options are not in or near the rotator cuff or shoulder. The nucleus pulposus is located in the center of each intervertebral disk. The articular processes are projections in each vertebral disk that lock onto the next vertebra, thereby stabilizing the spinal column. The medial epicondyle is located at the elbow.

A 40-year-old black man is in the office for his annual physical examination. Which statement regarding the PSA blood test is true? a. Should be performed at age 50 years. b. Should be performed with this visit. c. Should be performed at age 45 years. d. Is only necessary if a family history of prostate cancer exists.

ANS: C According to the American Cancer Society, the PSA blood test should be performed annually for black men beginning at age 45 years and annually for all other men over age 50 years.

During an examination of a 7-year-old girl, the nurse notices that the girl is showing breast budding. What should the nurse do next? a. Nothing; breast budding is a normal finding. b. Ask the young girl if her periods have started. c. Assess the girl's weight and body mass index (BMI). d. Ask the girl's mother at what age she started to develop breasts.

ANS: C Adolescent breast development usually begins between 8 and 10 years of age. However, research has shown that girls with overweight or obese BMI levels have a higher occurrence of early onset of breast budding (before age 8 years for black girls and age 10 years for white girls) and early menarche. Thus, the nurse should assess the patient's BMI. This is not a normal finding. Adolescent breast development usually begins between 8 and 10 years of age and precedes menarche by about 2 years.

The nurse is assessing a 1-week-old infant and is testing his muscle strength. The nurse lifts the infant with hands under the axillae and notices that the infant starts to "slip" between the hands. What does the nurse suspect? a. A fractured clavicle b. Possible deformity of the spine c. Weakness of the shoulder muscles d. This is a normal finding for an infant at this age

ANS: C An infant who starts to "slip" between the nurse's hands shows weakness of the shoulder muscles. An infant with normal muscle strength wedges securely between the nurse's hands. The other responses are not correct.

When taking the health history on a patient with a seizure disorder, the nurse assesses whether the patient has an aura. Which of these would be the best question for obtaining this information? a. "Does your muscle tone seem tense or limp?" b. "After the seizure, do you spend a lot of time sleeping?" c. "Do you have any warning sign before your seizure starts?" d. "Do you experience any color change or incontinence during the seizure?"

ANS: C Aura is a subjective sensation that precedes a seizure; it could be auditory, visual, or motor. The other questions do not solicit information about an aura.

What usually occurs to the cells in the reproductive tract to cause the changes normally associated with menopause? a. Aging b. Becoming fibrous c. Estrogen dependent d. Able to respond to progesterone

ANS: C Because cells in the reproductive tract are estrogen dependent, decreased estrogen levels during menopause bring dramatic physical changes. The other options are not correct.

A woman is in the family planning clinic seeking birth control information. She states that her breasts "change all month long" and that she is worried that this is unusual. What is the best response by the nurse? a. "Continual changes in your breasts are unusual. The breasts of nonpregnant women usually stay pretty much the same all month long." b. "Breast changes in response to stress are very common and you should assess your life for stressful events." c. "Because of the changing hormones during the monthly menstrual cycle, cyclic breast changes are common." d. "Breast changes normally occur only during pregnancy. You should get a pregnancy test done as soon as possible."

ANS: C Breasts of the nonpregnant woman change with the ebb and flow of hormones during the monthly menstrual cycle. During the 3 to 4 days before menstruation, the breasts feel full, tight, heavy, and occasionally sore. The breast volume is smallest on days 4 to 7 of the menstrual cycle.

A patient who has had rheumatoid arthritis for years comes to the clinic to ask about changes in her fingers. The nurse will assess for signs of what problems? a. Heberden nodes b. Bouchard nodules c. Swan-neck deformities d. Dupuytren contractures

ANS: C Changes in the fingers caused by chronic rheumatoid arthritis include swan-neck and boutonniere deformities. Heberden nodes and Bouchard nodules are associated with osteoarthritis. Dupuytren contractures of the digits occur because of chronic hyperplasia of the palmar fascia.

A patient states, "I can hear a crunching or grating sound when I kneel." She also states that "it is very difficult to get out of bed in the morning because of stiffness and pain in my joints." The nurse should assess for signs of what problem? a. Bone spur b. Tendonitis c. Crepitation d. Fluid in the knee joint

ANS: C Crepitation is an audible and palpable crunching or grating that accompanies movement and occurs when articular surfaces in the joints are roughened, as with rheumatoid arthritis. A bone spur is a bony projection (osteophyte) that develops along a bone edge, usually where bones meet at a joint. They often do not cause pain, but when they do, it is usually pain with movement in the specific joint with the bone spur. Tendonitis is an inflammation of a tendon and produces a swelling and tenderness to that one spot in the joint and affects only certain planes of ROM, especially during active ROM. Excess fluid in the knee can cause swelling and difficulty moving the knee, but usually does not cause pain, although the disease process causing the fluid (e.g. rheumatoid arthritis, osteoarthritis) may cause pain. The symptoms this patient is experiencing (audible and palpable crunching when kneeling indicates crepitation. Crepitation is an audible and palpable crunching or grating that accompanies movement and occurs when articular surfaces in the joints are roughened, as with rheumatoid arthritis.

The nurse is performing a genitourinary assessment on a 50-year-old obese male laborer. On examination, the nurse notices a painless round swelling close to the pubis in the area of the internal inguinal ring that is easily reduced when the individual is supine. What type of hernia do these findings suggest? a. Scrotal b. Femoral c. Direct inguinal d. Indirect inguinal

ANS: C Direct inguinal hernias occur most often in men over the age of 40 years. It is an acquired weakness brought on by heavy lifting, obesity, chronic cough, or ascites. The direct inguinal hernia is usually a painless, round swelling close to the pubis in the area of the internal inguinal ring that is easily reduced when the individual is supine. A scrotal hernia appears with an enlarged testis that does not transilluminate and may reduce when supine. It may be painful with straining. Upon palpation a soft, mushy mass which is distinct from the normal testis can be palpated and the palpating fingers cannot get above the mass. A femoral hernia usually presents with pain that is constant and may be severe and become strangulated. With indirect inguinal hernias, pain occurs with straining and a soft swelling increases with increased intra-abdominal pressure, which may decrease when the patient lies down.

After completing an assessment of a 60-year-old white male with a family history of colon cancer, the nurse discusses with him early detection measures for colon cancer. What should the nurse include in the instructions? a. Annual proctoscopy b. Annual PSA blood test c. Colonoscopy every 10 years d. Fecal occult blood test every 6 months

ANS: C Early detection measures for colon cancer include an annual fecal occult blood test after age 50 years (after age 45 for African Americans), a colonoscopy every 10 years after age 50, and a PSA blood test annually for men over 50 years old (45 years for African Americans).

The nurse suspects that a patient has carpal tunnel syndrome and wants to perform the Phalen test. What instructions should the nurse give the patient to perform this test? a. Dorsiflex the foot. b. Plantarflex the foot. c. Hold both hands back to back while flexing the wrists 90 degrees for 60 seconds. d. Hyperextend the wrists with the palmar surface of both hands touching, and wait for 60 seconds.

ANS: C For the Phalen test, the nurse should ask the person to hold both hands back to back while flexing the wrists 90 degrees. Acute flexion of the wrist for 60 seconds produces no symptoms in the normal hand. The Phalen test reproduces numbness and burning in a person with carpal tunnel syndrome. The other actions are not correct when testing for carpal tunnel syndrome.

A patient's annual physical examination reveals a lateral curvature of the thoracic and lumbar segments of his spine; however, this curvature disappears with forward bending. What is this abnormality called? a. Dislocated hip b. Structural scoliosis c. Functional scoliosis d. Herniated nucleus pulposus

ANS: C Functional scoliosis is flexible and apparent with standing but disappears with forward bending. Structural scoliosis is fixed; the curvature shows both when standing and when bending forward. These findings are not indicative of a herniated nucleus pulposus or dislocated hip.

A patient tells the nurse that, "All my life I've been called 'knock knees'." What is medical term for this condition? a. Genu varum b. Pes planus c. Genu valgum d. Metatarsus adductus

ANS: C Genu valgum is also known as knock knees and is present when more than 2.5 cm is between the medial malleoli when the knees are together. Pes planus, or flat foot, is pronation, or turning in, of the medial side of the foot. Metatarsus adductus is adduction, or turning inward, of the front half of the foot. The term used to describe knock knees is genu valgum. Genu valgum is present when more than 2.5 cm is between the medial malleoli when the knees are together.

Which is an accessory glandular structure for the male genital organs? a. Testis b. Scrotum c. Prostate d. Vas deferens

ANS: C Glandular structures accessory to the male genital organs are the prostate, seminal vesicles, and bulbourethral glands

A patient has had three pregnancies and two live births. How should the nurse record this information? a. G2; P2; AB1 b. G3; P2; AB0 c. G3; P2; AB1 d. G3; P3; AB1

ANS: C Gravida (G) is the number of pregnancies. Para (P) is the number of births. Abortions (AB) are interrupted pregnancies, including elective abortions and spontaneous miscarriages.

In examining a 70-year-old male patient, the nurse notices that he has bilateral gynecomastia. Which of the following describes the nurse's best course of action? a. Recommend that he make an appointment with his physician for a mammogram. b. Acknowledge it as benign breast enlargement which is not unusual in men. c. Explain that this condition may be the result of hormonal changes, and recommend that he see his physician. d. Explain that gynecomastia in men is usually associated with prostate enlargement and recommend that he be thoroughly screened.

ANS: C Gynecomastia may reappear in the aging man and may be attributable to a testosterone deficiency. The patient should see a physician to determine the cause and possible treatment. This is not considered a normal finding and a patient should see a physician to determine the cause and possible treatment. A mammogram is likely not necessary and it is not associated with prostate enlargement.

During the interview with a female patient, the nurse gathers data that indicates the patient is perimenopausal. Which of these statements made by this patient leads to this conclusion? a. "I have noticed that my muscles ache at night when I go to bed." b. "I will be very happy when I can stop worrying about having a period." c. "I have been noticing that I sweat a lot more than I used to, especially at night." d. "I have only been pregnant twice, but both times I had breast tenderness as my first symptom."

ANS: C Hormone shifts occur during the perimenopausal period, and associated symptoms of menopause may occur, such as hot flashes, night sweats, numbness and tingling, headache, palpitations, drenching sweats, mood swings, vaginal dryness, and itching. Muscle aches at night and breast tenderness as the first sign of pregnancy are not perimenopausal symptoms and the patient stating they will be happy to not have to worry about periods also does not indicate perimenopause.

The nurse is describing how to perform a testicular self-examination to a patient. Which statement is most appropriate? a. "A good time to examine your testicles is just before you take a shower." b. "The testicle is egg shaped and movable. It feels firm and has a lumpy consistency." c. "If you notice an enlarged testicle or a painless lump, call your health care provider." d. "Perform a testicular examination at least once a week to detect the early stages of testicular cancer."

ANS: C If the patient notices a firm painless lump, a hard area, or an overall enlarged testicle, then he should call his health care provider for further evaluation. The testicle normally feels rubbery with a smooth surface. A good time to examine the testicles is during the shower or bath, when one's hands are warm and soapy and the scrotum is warm. Testicular self-examination should be performed once a month

A 2-month-old uncircumcised infant has been brought to the clinic for a well-baby checkup. How would the nurse proceed with the genital examination? a. Elicit the cremasteric reflex. b. The glans is assessed for redness or lesions. c. Retracting the foreskin should be avoided until the infant is 3 months old. d. Any dirt or smegma that has collected under the foreskin should be noted.

ANS: C If uncircumcised, then the foreskin is normally tight during the first 3 months and should not be retracted because of the risk for tearing the membrane attaching the foreskin to the shaft. The cremasteric reflex (retracting the scrotal contents) is strong in infants and care should be taken not to elicit it. Since retracting the foreskin on an uncircumcised infant is not recommended until the infant is 3 months old, the glans and dirt and smegma under the foreskin cannot be assessed.

During an assessment of a 22-year-old woman who sustained a head injury from an automobile accident 4 hours earlier, the nurse notices the following changes: pupils were equal, but now the right pupil is fully dilated and nonreactive, and the left pupil is 4 mm and reacts to light. What do these findings suggest? a. Injury to the O.D. b. Test inaccurately performed c. Increased intracranial pressure d. Normal response after a head injury

ANS: C In a person with a brain injury, a sudden, unilateral, dilated, and nonreactive pupil is ominous. CN III runs parallel to the brainstem. When increasing intracranial pressure pushes down the brainstem (uncal herniation), it puts pressure on CN III, causing pupil dilation. The other responses are incorrect.

What structure secretes a thin, milky alkaline fluid to enhance the viability of sperm? a. Cowper gland b. Median sulcus c. Prostate gland d. Bulbourethral gland

ANS: C In men, the prostate gland secretes a thin milky alkaline fluid that enhances sperm viability. The Cowper glands (also known as bulbourethral glands) secrete a clear, viscid mucus. The median sulcus is a groove that divides the lobes of the prostate gland and does not secrete fluid.

During an examination of an aging man, the nurse recognizes that which finding is an expected or normal change? a. Enlarged scrotal sac b. Increased pubic hair c. Decreased penis size d. Increased rugae over the scrotum

ANS: C In the aging man, the amount of pubic hair decreases, the penis size decreases, and the rugae over the scrotal sac decreases. The scrotal sac does not enlarge.

During an examination, the nurse asks a patient to bend forward from the waist and notices that the patient has lateral tilting. When his leg is raised straight up, the patient states pain going down his buttock into his leg. What does the nurse suspect? a. Scoliosis b. Meniscus tear c. Herniated nucleus pulposus d. Spasm of paravertebral muscles

ANS: C Lateral tilting and sciatic pain with straight leg raising are findings that occur with a herniated nucleus pulposus. The other options are not correct.

The nurse is performing a neurologic assessment on a 41-year-old woman with a history of diabetes. When testing her ability to feel the vibrations of a tuning fork, the nurse notices that the patient is unable to feel vibrations on the great toe or ankle bilaterally, but she is able to feel vibrations on both patellae. Given this information, what would the nurse suspect? a. Hyperalgesia b. Hyperesthesia c. Peripheral neuropathy d. Lesion of sensory cortex

ANS: C Loss of vibration sense occurs with peripheral neuropathy (e.g., diabetes and alcoholism). Peripheral neuropathy is worse at the feet and gradually improves as the examiner moves up the leg, as opposed to a specific nerve lesion, which has a clear zone of deficit for its dermatome. The other responses are incorrect.

A 2-year-old boy has been diagnosed with physiologic cryptorchidism. Considering this diagnosis, what will the nurse most likely observe during the assessment? a. Testes that are hard and painful to palpation b. Atrophic scrotum and a bilateral absence of the testis c. Absence of the testis in the scrotum, but the testis can be milked down d. Testes that migrate into the abdomen when the child squats or sits cross-legged

ANS: C Migratory testes (physiologic cryptorchidism) are common because of the strength of the cremasteric reflex and the small mass of the prepubertal testes. The affected side has a normally developed scrotum and the testis can be milked down. The other responses are not correct.

START OF MUSCULOSKELETAL SYSTEM QUESTIONS A patient is being assessed for range-of-joint movement. The nurse asks him to move his arm in toward the center of his body. What is this movement called? a. Flexion b. Abduction c. Adduction d. Extension

ANS: C Moving a limb toward the midline of the body is called adduction; moving a limb away from the midline of the body is called abduction. Flexion is bending a limb at a joint; and extension is straightening a limb at a joint.

The nurse is examining a 35-year-old female patient. During the health history, the nurse notices that she has had two term pregnancies, and both babies were delivered vaginally. During the internal examination, the nurse observes that the cervical os is a horizontal slit with some healed lacerations and that the cervix has some nabothian cysts that are small, smooth, and yellow. In addition, the nurse notices that the cervical surface is granular and red, especially around the os. Finally, the nurse notices the presence of stringy, opaque, odorless secretions. Which of these findings are abnormal? a. Nabothian cysts are present. b. The cervical os is a horizontal slit. c. The cervical surface is granular and red. d. Stringy and opaque secretions are present.

ANS: C Normal findings: Nabothian cysts may be present on the cervix after childbirth. The cervical os is a horizontal, irregular slit in the parous woman. Secretions vary according to the day of the menstrual cycle, and may be clear and thin or thick, opaque, and stringy. The surface is normally smooth, but cervical eversion, or ectropion, may occur where the endocervical canal is rolled out. Abnormal finding: The cervical surface should not be reddened or granular, which may indicate a lesion.

A patient contacts the office and tells the nurse that she is worried about her 10-year-old daughter having breast cancer. She describes a unilateral enlargement of the right breast with associated tenderness. She is worried because the left breast is not enlarged. Which response by the nurse to the mother is best? a. "Breast development is usually fairly symmetric your daughter should be examined right away." b. "You should bring in your daughter right away because breast cancer is fairly common in preadolescent girls." c. "Although an examination of your daughter would rule out a problem, her breast development is most likely normal." d. "It is unusual for breasts that are first developing to feel tender because they haven't developed much fibrous tissue."

ANS: C Occasionally, one breast may grow faster than the other, producing a temporary asymmetry, which may cause some distress; reassurance is necessary. Tenderness is also common.

When the nurse is interviewing a preadolescent girl, which opening question would be least threatening? a. "Do you have any questions about growing up?" b. "What has your mother told you about growing up?" c. "When did you notice that your body was changing?" d. "I remember being very scared when I got my period. How do you think you'll feel?"

ANS: C Open-ended questions such as, "When did you...?" rather than "Do you...?" should be asked. Open-ended questions are less threatening because they imply that the topic is normal and unexceptional.

A 14-year-old boy who has been diagnosed with Osgood-Schlatter disease reports painful swelling just below the knee for the past 5 months. Which response by the nurse is appropriate? a. "If these symptoms persist, you may need arthroscopic surgery." b. "You are experiencing degeneration of your knee, which may not resolve." c. "Your disease is due to repeated stress on the patellar tendon. It is usually self-limited, and your symptoms should resolve with rest." d. "Increasing your activity and performing knee-strengthening exercises will help decrease the inflammation and maintain mobility in the knee."

ANS: C Osgood-Schlatter disease is a painful swelling of the tibial tubercle just below the knee and most likely due to repeated stress on the patellar tendon. It is usually self-limited, occurring during rapid growth and most often in boys. The symptoms resolve with rest. The other responses are not appropriate.

During a bimanual examination, the nurse detects a solid tumor on the ovary that is heavy and fixed, with a poorly defined mass. What does this finding suggest? a. Ovarian cyst b. Endometriosis c. Ovarian cancer d. Ectopic pregnancy

ANS: C Ovarian tumors that are solid, heavy, and fixed, with poorly defined mass are suggestive of malignancy. Benign masses may feel mobile and solid. An ovarian cyst may feel smooth, round, fluctuant, mobile, and nontender. With an ectopic pregnancy, the examiner may feel a palpable, tender pelvic mass that is solid, mobile, and unilateral. Endometriosis may have masses (in various locations in the pelvic area) that are small, firm, nodular, and tender to palpation, with enlarged ovaries.

The nurse is caring for a patient who has just had neurosurgery. To assess for increased intracranial pressure, what would the nurse include in the assessment? a. CNs, motor function, and sensory function b. Deep tendon reflexes, vital signs, and coordinated movements c. Level of consciousness, motor function, pupillary response, and vital signs d. Mental status, deep tendon reflexes, sensory function, and pupillary response

ANS: C People who have a neurologic deficit from a systemic disease process, head trauma, or neurosurgery are at increased risk for developing increased intracranial pressure. These people must be closely monitored for any improvement or deterioration in neurologic status. The nurse should use an abbreviation of the neurologic examination in the following sequence: level of consciousness, motor function, pupillary response, and vital signs.

The nurse is teaching a class on preventing osteoporosis to a group of perimenopausal women. Which of these actions is the best way to prevent or delay bone loss in this group? a. Assessing bone density annually b. Taking medications to prevent osteoporosis c. Performing physical activity, such as fast walking d. Taking 800 mg calcium and 200 IU vitamin D supplements daily

ANS: C Physical activity, such as fast walking, delays or prevents bone loss in perimenopausal women. The faster the pace of walking, the higher the preventive effect is on the risk for hip fracture. The other options are not correct. Annually assessing bone density does not prevent or delay bone loss, it just monitors it. There are no medications to prevent osteoporosis, but to treat it. Taking 800 mg calcium and 200 IU vitamin D supplements daily is not enough to meet the recommended daily doses for a perimenopausal woman. The best way to prevent or delay bone loss is exercise.

During a neonatal examination, the nurse notices that the newborn infant has six toes. How should the nurse document this finding? a. Unidactyly b. Syndactyly c. Polydactyly d. Multidactyly

ANS: C Polydactyly is the presence of extra fingers or toes. Syndactyly is webbing between adjacent fingers or toes. The other terms are not correct.

During the interview, a patient reveals that she has some vaginal discharge. She is worried that it may be a sexually transmitted infection. What would be the most appropriate response by the nurse? a. "Oh, don't worry. Some cyclic vaginal discharge is normal." b. "Have you been engaging in unprotected sexual intercourse?" c. "I'd like more information about the discharge. What color is it?" d. "Have you had any urinary incontinence associated with the discharge?"

ANS: C Questions that help the patient reveal more information about her symptoms should be asked in a nonthreatening manner. Asking about the amount, color, and odor of the vaginal discharge provides the opportunity for further assessment. Normal vaginal discharge is small, clear or cloudy, and always nonirritating

A patient is reporting pain in his joints that is worse in the morning, better after he moves around for a while, and then gets worse again if he sits for long periods. The nurse should assess for other signs of what problem? a. Tendinitis b. Osteoarthritis c. Rheumatoid arthritis d. Intermittent claudication

ANS: C Rheumatoid arthritis pain is worse in the morning when a person arises and then improves with movement. Movement increases most other types of joint pain.

A patient is able to flex his right arm forward without difficulty or pain but is unable to abduct his arm because of pain and muscle spasms. What does the nurse suspect? a. Crepitation b. Rheumatoid arthritis c. Rotator cuff lesions d. A dislocated shoulder

ANS: C Rotator cuff lesions may limit range of motion and cause pain and muscle spasms during abduction, whereas forward flexion remains fairly normal. Crepitation is an audible and palpable crunching or grating that accompanies movement and occurs when articular surfaces in the joints are roughened, as with rheumatoid arthritis. Rheumatoid arthritis is a chronic inflammatory pain condition in the joints. Joint involvement is symmetric and bilateral, with heat, redness, swelling, and painful motion of affected joints. A dislocated shoulder shows an obvious deformity and severe pain with movement. Crepitation is an audible and palpable crunching or grating that accompanies movement and occurs when articular surfaces in the joints are roughened, as with rheumatoid arthritis. Rheumatoid arthritis is a chronic inflammatory pain condition in the joints. Joint involvement is symmetric and bilateral (not just one side as in this patient), with heat, redness, swelling, and painful motion of affected joints. A dislocated shoulder shows an obvious deformity and severe pain with movement (not just with certain movements as with this patient). The symptoms this patient is experiencing are that of rotator cuff lesions.

A woman states that 2 weeks ago she had a urinary tract infection that was treated with an antibiotic. What should the nurse ask the woman? a. "Have you had excessive vaginal bleeding?" b. "Have you experienced changes in your urination patterns?" c. "Do you have any unusual vaginal discharge or itching?" d. "Have you noticed any changes in your desire for intercourse?"

ANS: C Several medications may increase the risk for vaginitis. Broad-spectrum antibiotics alter the balance of normal flora, which may lead to the development of vaginitis. The other questions are not appropriate.

During an assessment of an 80-year-old patient, the nurse notices the following: an inability to identify vibrations at her ankle and to identify the position of her big toe, a slower and more deliberate gait, and a slightly impaired tactile sensation. All other neurologic findings are normal. How should the nurse interpret these findings? a. CNS dysfunction b. Lesion in the cerebral cortex c. Normal changes attributable to aging d. Demyelination of nerves attributable to a lesion

ANS: C Some aging adults show a slower response to requests, especially for those calling for coordination of movements. The findings listed are normal in the absence of other significant abnormal findings. The other responses are incorrect.

The nurse notices that a patient has had a pale, yellow, greasy stool, or steatorrhea. What is the cause of this finding? a. Occult bleeding b. Absent bile pigment c. Increased fat content d. Ingestion of bismuth preparations

ANS: C Steatorrhea (pale, yellow, greasy stool) is caused by increased fat content in the stools, as in malabsorption syndrome. Occult bleeding and ingestion of bismuth products cause a black stool, and absent bile pigment causes a gray-tan stool.

When the nurse is performing a testicular examination on a 25-year-old man, which finding is considered normal? a. Nontender subcutaneous plaques b. Scrotal area that is dry, scaly, and nodular c. Testes that feel oval and movable and are slightly sensitive to compression d. Single, hard, circumscribed, movable mass, less than 1 cm under the surface of the testes

ANS: C Testes normally feel oval, firm and rubbery, smooth, and bilaterally equal and are freely movable and slightly tender to moderate pressure. The scrotal skin should not be dry, scaly, or nodular or contain subcutaneous plaques. Any mass would be an abnormal finding.

While assessing a 7-month-old infant, the nurse makes a loud noise and notices the following response: abduction and flexion of the arms and legs; fanning of the fingers, and curling of the index finger and thumb in a C position, followed by the infant bringing in the arms and legs to the body. What does the nurse recall about this response? a. This response could indicate brachial nerve palsy. b. This reaction is an expected startle response at this age. c. This reflex should have disappeared between 1 and 4 months of age. d. This response is normal as long as the movements are bilaterally symmetric.

ANS: C The Moro reflex is present at birth and usually disappears at 1 to 4 months. Absence of the Moro reflex in the newborn or its persistence after 5 months of age indicates severe central nervous system injury. The other responses are incorrect.

What is the articulation of the mandible and the temporal bone called? a. Intervertebral foramen b. Condyle of the mandible c. Temporomandibular joint d. Zygomatic arch of the temporal bone

ANS: C The articulation of the mandible and the temporal bone is the temporomandibular joint. The other responses are not correct.

The nurse is conducting a class on BSE. Which of these statements indicates the proper BSE technique? a. The best time to perform BSE is in the middle of the menstrual cycle. b. A woman should perform BSEs bimonthly unless she has fibrocystic breast tissue. c. The best time to perform a BSE is 4 to 7 days after the first day of the menstrual period. d. If she suspects that she is pregnant, then the woman should not perform a BSE until after her baby is born.

ANS: C The best time to conduct a BSE is right after the menstrual period, or the fourth through seventh day of the menstrual cycle, when the breasts are the smallest and least congested. The pregnant or menopausal woman who is not having menstrual periods should be advised to select a familiar date to examine her breasts each month—for example, her birth date or the day the rent is due. Women do not need to be advised to perform BSEs bimonthly.

In performing an assessment of a woman's axillary lymph system, the nurse should assess which of these nodes? a. Central, axillary, lateral, and sternal b. Pectoral, lateral, anterior, and sternal c. Central, lateral, pectoral, and subscapular d. Lateral, pectoral, axillary, and suprascapular

ANS: C The breast has extensive lymphatic drainage, but there are no sternal or suprascapular lymph nodes. The four groups of axillary nodes are the (1) central, (2) pectoral (anterior), (3) subscapular (posterior), and (4) lateral.

A 30-year-old woman tells the nurse that she has been very unsteady and has had difficulty in maintaining her balance. Which area of the brain most concerns the nurse? a. Thalamus b. Brainstem c. Cerebellum d. Extrapyramidal tract

ANS: C The cerebellar system coordinates movement, maintains equilibrium, and helps maintain posture. So the nurse would be most concerned about this area of the brain. The thalamus is the primary relay station where sensory pathways of the spinal cord, cerebellum, and brainstem form synapses on their way to the cerebral cortex. The brainstem consists of the midbrain, pons, and medulla and has various functions, especially concerning autonomic centers. The extrapyramidal tract maintains muscle tone for gross automatic movements, such as walking. The thalamus is the primary relay station where sensory pathways of the spinal cord, cerebellum, and brainstem form synapses on their way to the cerebral cortex. The brainstem consists of the midbrain, pons, and medulla and has various functions, especially concerning autonomic centers. The extrapyramidal tract maintains muscle tone for gross automatic movements, such as walking. With this patient's unsteady gait and balance problems, the nurse would be most concerned with the cerebellum.

The nurse should test the functioning of which structure(s) when determining whether a person is oriented to his or her surroundings? a. Cerebellum b. Cranial nerves c. Cerebral cortex d. Medulla oblongata

ANS: C The cerebral cortex (the outer layer of the cerebrum) is responsible for thought, memory, reasoning, sensation, and voluntary movement. Thus, determining orientation would assess the functioning of the cerebral cortex. The cerebellum is a coiled structure located under the occipital lobe that is concerned with motor coordination of voluntary movements, equilibrium, and muscle tone, but not a person's thought processes or orientation. The cranial nerves are responsible for relaying sensory and motor information to and from the brain, but are not involved in thought processes or orientation. The medulla oblongata is located in the brainstem and it has vital autonomic (involuntary) centers (respiration, heart, gastrointestinal function) and nuclei of cranial nerves VIII through XII. Pyramidal decussation (crossing of the motor fibers) also occurs here, but the medulla oblongata is not involved in thought or orientation.

During an examination, how would the nurse expect the cervical os of a woman who has never had children to appear? a. Everted b. Stellate c. Small and round d. As a horizontal irregular slit

ANS: C The cervical os in a nulliparous woman is small and round. In the parous woman, it is a horizontal, irregular slit that also may show healed lacerations on the sides. It does not appear stellate (resembling a star shape) or everted (rolled out).

A 62-year-old man is experiencing fever, chills, malaise, urinary frequency, and urgency. He also reports urethral discharge and a dull aching pain in the perineal and rectal area. These symptoms are most consistent with which condition? a. BPH b. Polyps c. Prostatitis d. Carcinoma of the prostate

ANS: C The common presenting symptoms of prostatitis are fever, chills, malaise, and urinary frequency and urgency. The individual may also have dysuria, urethral discharge, and a dull aching pain in the perineal and rectal area. The signs and symptoms of benign prostatic hypertrophy (BPH) include urinary frequency, urgency, hesitancy, straining to urinate, weak stream, intermittent stream, sensation of incomplete emptying, and nocturia. The prostate surface feels smooth, rubbery, or firm (like the consistency of the nose), with the median sulcus obliterated. A rectal polyp is a protruding growth from the rectal mucous membrane and is fairly common. Rectal polyps are difficult to palpate and are found on examination with a scope. The signs and symptoms of carcinoma of the prostate include frequency, nocturia, hematuria, weak stream, hesitancy, pain or burning on urination, and continuous pain in lower back, pelvis, and thighs. It often starts as a single hard nodule on the posterior surface of the prostate and as it invades normal tissue, multiple hard nodules appear, or the entire gland feels stone-hard and fixed and the median sulcus obliterated.

The nurse is preparing for an internal genitalia examination of a woman. Which order of the examination is correct? a. Bimanual, speculum, and rectovaginal b. Speculum, rectovaginal, and bimanual c. Speculum, bimanual, and rectovaginal d. Rectovaginal, bimanual, and speculum

ANS: C The correct sequence is speculum examination, then bimanual examination after removing the speculum, and then rectovaginal examination. The examiner should change gloves before performing the rectovaginal examination to avoid spreading any possible infection.

The nurse is examining the glans and knows which finding is normal for this area? a. Hair is without pest inhabitants. b. The skin is wrinkled and without lesions. c. Smegma may be present under the foreskin of an uncircumcised male. d. The meatus may have a slight discharge when the glans is compressed.

ANS: C The glans looks smooth and without lesions and does not have hair. The meatus should not have any discharge when the glans is compressed. Some cheesy smegma may have collected under the foreskin of an uncircumcised male.

The nurse is examining the hip area of a patient and palpates a flat depression on the upper, lateral side of the thigh when the patient is standing. What is the nurse palpating? a. Iliac crest b. Ischial tuberosity c. Greater trochanter d. Gluteus maximus muscle

ANS: C The greater trochanter of the femur is palpated when the person is standing, and it appears as a flat depression on the upper lateral side of the thigh. The iliac crest is the upper part of the hip bone; the ischial tuberosity lies under the gluteus maximus muscle and is palpable when the hip is flexed; and the gluteus muscle is part of the buttocks. The iliac crest is the upper part of the hip bone (not lateral); the ischial tuberosity lies under the gluteus maximus muscle and is palpable when the hip is flexed (not standing); and the gluteus muscle is part of the buttocks. The flat depression in the upper lateral side of the thigh that the nurse is palpating is the greater trochanter.

The nurse is examining only the rectal area of a woman and should place the woman in what position? a. Prone b. Lithotomy c. Left lateral decubitus d. Bending over the table while standing

ANS: C The left lateral decubitus position is used when examining only the rectal area. If the genitalia are also going to be examined, the nurse should place the female patient in the lithotomy.

During a physical examination, a 45-year-old woman states that she has had a crusty, itchy rash on her breast for approximately 2 weeks. In trying to find the cause of the rash, which question would be important for the nurse to ask? a. "Is the rash raised and red?" b. "Does it appear to be cyclic?" c. "Where did the rash first appear—on the nipple, the areola, or the surrounding skin?" d. "What was she doing when she first noticed the rash, and do her actions make it worse?"

ANS: C The location where the rash first appeared is important for the nurse to determine. Paget disease starts with a small crust on the nipple apex and then spreads to the areola. Eczema or other dermatitis rarely starts at the nipple unless it is a result of breastfeeding. It usually starts on the areola or surrounding skin and then spreads to the nipple.

While palpating the prostate gland through the rectum, which finding would the nurse recognize as abnormal? a. Heart shaped b. Palpable central groove c. Tenderness to palpation d. Elastic and rubbery consistency

ANS: C The normal prostate gland should feel smooth, elastic, and rubbery; slightly movable; heart-shaped with a palpable central groove; and not be tender to palpation.

During the physical examination, the nurse notices that a female patient has an inverted left nipple. Which statement regarding this is most accurate? a. Normal nipple inversion is usually bilateral. b. Unilateral inversion of a nipple is always a serious sign. c. Whether the inversion is a recent change should be determined. d. Nipple inversion is not significant unless accompanied by an underlying palpable mass.

ANS: C The nurse should distinguish between a recently retracted nipple from one that has been inverted for many years or since puberty. Normal nipple inversion may be unilateral or bilateral and usually can be pulled out; that is, if it is not fixed. Recent nipple retraction signifies acquired disease (see Table 17-3).

During the taking of the health history of a 78-year-old man, his wife states that he occasionally has problems with short-term memory loss and confusion: "He can't even remember how to button his shirt." When assessing his sensory system, which action by the nurse is most appropriate? a. The nurse would perform the tests, knowing that mental status does not affect sensory ability. b. The nurse would proceed with an explanation of each test, making certain that the wife understands. c. Before testing, the nurse would assess the patient's mental status and ability to follow directions. d. The nurse would not test the sensory system as part of the examination because the results would not be valid.

ANS: C The nurse should ensure the validity of the sensory system testing by making certain that the patient is alert, cooperative, comfortable, and has an adequate attention span. Otherwise, the nurse may obtain misleading and invalid results.

A 50-year-old woman is in the clinic for weakness in her left arm and leg that she has noticed for the past week. The nurse should perform which type of neurologic examination? a. Glasgow Coma Scale b. Neurologic recheck examination c. Complete neurologic examination d. Screening neurologic examination

ANS: C The nurse should perform a complete neurologic examination on an individual who has neurologic concerns (e.g., headache, weakness, loss of coordination) or who is showing signs of neurologic dysfunction. The Glasgow Coma Scale is used to define a person's level of consciousness. The neurologic recheck examination is appropriate for those who are demonstrating neurologic deficits. The screening neurologic examination is performed on seemingly well individuals who have no significant subjective findings from the health history.

While assessing a hospitalized patient who is jaundiced, the nurse notices that the patient has been incontinent of stool. The stool is loose and gray-tan in color. What does this finding indicate? a. Occult blood b. Inflammation c. Absent bile pigment d. Ingestion of iron preparations

ANS: C The presence of gray-tan stool indicates the absence of bile pigment, which can occur with obstructive jaundice. The ingestion of iron preparations and the presence of occult blood turn the stools to a black color. Jellylike mucus shreds mixed in the stool would indicate inflammation

A patient with a lack of oxygen to his heart will have pain in his chest and possibly in the shoulder, arms, or jaw. The nurse knows that the best explanation why this occurs is which one of these statements? a. A problem exists with the sensory cortex and its ability to discriminate the location. b. The lack of oxygen in his heart has resulted in decreased amount of oxygen to the areas experiencing the pain. c. The sensory cortex does not have the ability to localize pain in the heart; consequently, the pain is felt elsewhere. d. A lesion has developed in the dorsal root, which is preventing the sensation from being transmitted normally.

ANS: C The sensory cortex is arranged in a specific pattern, forming a corresponding map of the body. Pain in the right hand is perceived at a specific spot on the map. Some organs, such as the heart, liver, and spleen, are absent from the brain map. Pain originating in these organs is referred because no felt image exists in which to have pain. Pain is felt by proxy, that is, by another body part that does have a felt image. The other responses are not correct explanations.

A 46-year-old man requires an assessment of his sigmoid colon. Which instrument or technique is most appropriate for this examination? a. Ultrasound b. Proctoscope c. Colonoscope d. Rectal examination with an examining finger

ANS: C The sigmoid colon is 40 cm long, and it is accessible to examination only with a colonoscope. The other responses are not appropriate for an examination of the entire sigmoid colon.

The area of the nervous system that responsible for mediating reflexes? a. Medulla b. Cerebellum c. Spinal cord d. Cerebral cortex

ANS: C The spinal cord is the main highway for ascending and descending fiber tracts that connect the brain to the spinal nerves; it is responsible for mediating reflexes. The medulla is the continuation of the spinal cord in the brain that contains all ascending and descending fiber tracts. Pyramidal decussation (crossing of the motor fibers) occurs here. The cerebellum is a coiled structure located under the occipital lobe that is concerned with motor coordination of voluntary movements, equilibrium, and muscle tone. The cerebral cortex is the outer layer of nerve cell bodies and is the center for a human's highest functions, governing thought, memory, reasoning, sensation, and voluntary movement.

While gathering equipment after an injection, a nurse accidentally received a prick from an improperly capped needle. To interpret this sensation, which of these areas must be intact? a. Corticospinal tract, medulla, and basal ganglia b. Pyramidal tract, hypothalamus, and sensory cortex c. Lateral spinothalamic tract, thalamus, and sensory cortex d. Anterior spinothalamic tract, basal ganglia, and sensory cortex

ANS: C The spinothalamic tract contains sensory fibers that transmit the sensations of pain, temperature, and crude or light touch. Fibers carrying pain and temperature sensations ascend the lateral spinothalamic tract, whereas the sensations of crude touch form the anterior spinothalamic tract. At the thalamus, the fibers synapse with another sensory neuron, which carries the message to the sensory cortex for full interpretation. The other options are not correct.

The nurse is performing an assessment on a 29-year-old woman who visits the clinic reporting "always dropping things and falling down." While testing rapid alternating movements, the nurse notices that the woman is unable to pat both of her knees. Her response is extremely slow and she frequently misses. What should the nurse suspect? a. Lesion of CN IX b. Vestibular disease c. Dysfunction of the cerebellum d. Inability to understand directions

ANS: C The symptoms this patient has been experiencing indicate dysfunction of the cerebellum. The cerebellum is concerned with motor coordination of voluntary movements, equilibrium, and muscle tone. When a person tries to perform rapid, alternating movements, responses that are slow, clumsy, and sloppy are indicative of cerebellar disease. Vestibular disease causes problems with balance and vertigo. Lesions of CN IX cause problems swallowing or gagging. Inability to understand directions would be r/t a problem in Wernicke's area in the brain and is not associated with dropping things or falling down.

An 18-year-old patient is having her first pelvic examination. Which action by the nurse is appropriate? a. Inviting her mother to be present during the examination. b. Avoiding the lithotomy position for this first time because it can be uncomfortable and embarrassing. c. Raising the head of the examination table and giving her a mirror so that she can view the examination. d. Fully draping her, leaving the drape between her legs elevated to avoid embarrassing her with eye contact.

ANS: C The techniques of the educational or mirror pelvic examination should be used. This is a routine examination with some modifications in attitude, position, and communication. First, the woman is considered an active participant, one who is interested in learning and in sharing decisions about her own health care. The woman props herself up on one elbow, or the head of the table is raised. Her other hand holds a mirror between her legs, above the examiner's hands. The young woman can see all that the examiner is doing and has a full view of her genitalia. The mirror works well for teaching normal anatomy and its relationship to sexual behavior. The examiner can ask her if she would like to have a family member, friend, or chaperone present for the examination. The drape should be pushed down between the patient's legs so that the nurse can see her face.

To palpate the temporomandibular joint, where should the nurse place his or her fingers? a. The depression inferior to the tragus of the ear b. The depression superior to the tragus of the ear c. The depression anterior to the tragus of the ear d. The depression posterior to the tragus of the ear

ANS: C The temporomandibular joint can be felt in the depression anterior to the tragus of the ear. The other locations are not correct.

While examining a 75-year-old woman, the nurse notices that the skin over her right breast is thickened and the hair follicles are exaggerated. What is this condition called? a. Dimpling b. Retraction c. Peau d'orange d. Benign breast disease

ANS: C This condition is known as peau d'orange. Lymphatic obstruction produces edema, which thickens the skin and exaggerates the hair follicles. The skin has a pig-skin or orange-peel appearance. This condition suggests cancer. Dimpling, or a pucker, indicates skin retraction. Retraction signs are caused by fibrosis in the breast tissue, usually caused by growing neoplasms. The fibrosis shortens with time, causing contrasting signs with the normally loose breast tissue. Benign breast disease is multiple tender masses that occur with numerous symptoms and physical findings such as swelling and tenderness, nodularity, dominant lumps, nipple discharge, and infections or inflammation. The symptoms this patient has indicate peau d'orange. In peau d'orange, lymphatic obstruction produces edema, which thickens the skin and exaggerates the hair follicles. The skin has a pig-skin or orange-peel appearance. This condition suggests cancer.

During an assessment of the newborn, the nurse expects to see which finding when the anal area is slightly stroked? a. Jerking of the legs b. Flexion of the knees c. Quick contraction of the sphincter d. Relaxation of the external sphincter

ANS: C To assess sphincter tone, the nurse should check the anal reflex by gently stroking the anal area and noticing a quick contraction of the sphincter. The other responses are not correct.

The nurse is examining a 2-month-old infant and notices asymmetry of the infant's gluteal folds. The nurse should assess for other signs of what disorder? a. Spina bifida b. Down syndrome c. Hip dislocation d. Fractured clavicle

ANS: C Unequal gluteal folds may accompany hip dislocation after 2 to 3 months of age, but some asymmetry may occur in healthy children. Further assessment is needed. The other responses are not correct.

The nurse is testing superficial reflexes on an adult patient. When stroking up the lateral side of the sole and across the ball of the foot, the nurse notices the plantar flexion of the toes. How should the nurse document this finding? a. Positive Babinski sign b. Plantar reflex abnormal c. Plantar reflex present d. Plantar reflex 2+ on a scale from "0 to 4+"

ANS: C With a reflex hammer, the nurse should draw a light stroke up the lateral side of the sole of the foot and across the ball of the foot, similar to an upside-down J. The normal response is plantar flexion of the toes and sometimes of the entire foot. A positive Babinski sign is abnormal and occurs with the response of dorsiflexion of the big toe and fanning of all toes. The plantar reflex is not graded on a 0 to 4+ scale.

The nurse is performing a digital examination of a patient's prostate gland and notices that a normal prostate gland includes which of the following characteristics? (Select all that apply.) a. Fixed mobility b. Boggy with a soft consistency c. 1 cm protrusion into the rectum d. Flat shape with no palpable groove e. Heart-shaped with a palpable central groove f. Smooth surface, elastic, and rubbery consistency

ANS: C, E, F The size of a normal prostate gland should be 2.5 cm long by 4 cm wide and should not protrude more than 1 cm into the rectum. The prostate should be heart-shaped, with a palpable central groove, a smooth surface, and elastic with a rubbery consistency. Abnormal findings include a flat shape with no palpable groove, boggy with a soft consistency, and fixed mobility

What is a common assessment finding in a boy younger than 2 years old? a. Inflamed and tender spermatic cord b. Presence of a hernia in the scrotum c. Penis that looks large in relation to the scrotum d. Presence of a hydrocele, or fluid in the scrotum

ANS: D A common scrotal finding in boys younger than 2 years of age is a hydrocele, or fluid in the scrotum. The other options are not correct.

A young swimmer comes to the sports clinic complaining of a very sore shoulder. He was running at the pool, slipped on some wet concrete, and tried to catch himself with his outstretched hand. He landed on his outstretched hand and has not been able to move his shoulder since. What does the nurse suspect? a. Joint effusion b. Tear of rotator cuff c. Adhesive capsulitis d. Dislocated shoulder

ANS: D A dislocated shoulder occurs with trauma involving abduction, extension, and external rotation (e.g., falling on an outstretched arm or diving into a pool). Joint effusion is swelling from excess fluid in the joint capsule. Tear of rotator cuff typically presents in a "hunched" position and limited abduction of arm. Adhesive capsulitis (frozen shoulder) presents with stiffness; progressive limitation of motion in abduction and external rotation, and unable to reach overhead; and pain caused by the formation of fibrous tissues in the joint capsule. Joint effusion is swelling from excess fluid in the joint capsule. Tear of rotator cuff typically presents in a "hunched" position and limited abduction of arm. Adhesive capsulitis (frozen shoulder) presents with stiffness; progressive limitation of motion in abduction and external rotation, and unable to reach overhead; and pain caused by the formation of fibrous tissues in the joint capsule. This patient appears to have a dislocated shoulder.

While performing a rectal examination, the nurse notices a firm, irregularly shaped mass. What should the nurse do next? a. Continue with the examination, and document the finding in the chart. b. Instruct the patient to return for a repeat assessment in 1 month. c. Tell the patient that a mass was felt, but it is nothing to worry about. d. Report the finding, and refer the patient to a specialist for further examination.

ANS: D A firm or hard mass with an irregular shape or rolled edges may signify carcinoma. Any mass that is discovered should be promptly reported for further examination. The other responses are not correct.

A 62-year-old man states that his physician told him that he has an "inguinal hernia." He asks the nurse to explain what a hernia is. Which response by the nurse is best? a. "Don't worry, most men your age develop hernias." b. "A hernia is often the result of a prenatal growth abnormality." c. "You should talk to your physician since he or she made the initial diagnosis." d. "A hernia is a loop of bowel protruding through a weak spot in the abdominal muscles."

ANS: D A hernia is a loop of bowel protruding through a weak spot in the musculature of the abdominal wall. It is not a result of a prenatal growth abnormality. Although the patient may need to talk to the physician who diagnosed the hernia, the nurse should still answer his question and should not tell him not to worry, but acknowledge his concerns. A hernia is not a result of a prenatal growth abnormality. The nurse should explain to him that a hernia is a loop of bowel protruding through a weak spot in the musculature of the abdominal wall.

Which of these statements about the peripheral nervous system is correct? a. The CNs enter the brain through the spinal cord. b. Efferent fibers carry sensory input to the central nervous system through the spinal cord. c. The peripheral nerves are inside the central nervous system and carry impulses through their motor fibers. d. The peripheral nerves carry input to the central nervous system by afferent fibers and away from the central nervous system by efferent fibers

ANS: D A nerve is a bundle of fibers outside of the central nervous system. The peripheral nerves carry input to the central nervous system by their sensory afferent fibers and deliver output from the central nervous system by their efferent fibers. The other responses are not r/t the peripheral nervous system.

During an assessment of a 20-year-old man, the nurse finds a small palpable lesion with a tuft of hair located directly over the coccyx. What does this finding indicate? a. Carcinoma b. Rectal polyp c. Pruritus ani d. Pilonidal cyst

ANS: D A pilonidal cyst or sinus is a hair-containing cyst or sinus located in the midline over the coccyx or lower sacrum. It often opens as a dimple with a visible tuft of hair and, possibly, an erythematous halo. Carcinoma, or a malignant neoplasm, in the colon or rectum is typically asymptomatic. Rectal polyps are protruding growths from the rectal mucous membrane and are fairly common. They are difficult to palpate and found on examination with a scope. Pruritus ani is intense itching and burning in the perineum and has several causes such as soaps, fecal soiling or hemorrhoids, sexually transmitted infections, and pinworm manifestations in children

The nurse has completed the musculoskeletal examination of a patient's knee and has found a positive bulge sign. How does the nurse interpret this finding? a. Irregular bony margins b. Soft-tissue swelling in the joint c. Swelling from fluid in the epicondyle d. Swelling from fluid in the suprapatellar pouch

ANS: D A positive bulge sign confirms the presence of swelling caused by fluid in the suprapatellar pouch. The other options are not correct.

A 14-year-old girl is anxious about not having reached menarche. When taking the health history, the nurse should ascertain which of the following? a. The age her mother developed breasts b. The age she began to develop pubic hair c. The age she began developing axillary hair d. The age the girl began to develop breasts

ANS: D According to Tanner's five stages of breast development, full development from stage 2 to stage 5 takes an average of 3 years, although the range is 11 2 to 6 years. Pubic hair develops during this time, and axillary hair appears 2 years after the onset of pubic hair. The beginning of breast development precedes menarche by approximately 2 years. Menarche occurs in breast development stage 3 or 4, usually just after the peak of the adolescent growth spurt, which occurs around age 12 years.

A 9-year-old girl is in the clinic for a sport physical examination. After some initial shyness she finally asks, "Am I normal? I don't seem to need a bra yet, but I have some friends who do. What if I never get breasts?" Which response by the nurse is best? a. "Don't worry, you still have plenty of time to develop." b. "I know just how you feel, I was a late bloomer myself. Just be patient, and they will grow." c. "You will probably get your periods before you notice any significant growth in your breasts." d. "I understand that it is hard to feel different

ANS: D Adolescent breast development usually begins between 8 and 10 years of age. The nurse should not belittle the girl's feelings by using statements like "don't worry" or by sharing personal experiences. The beginning of breast development precedes menarche by approximately 2 years.

A 65-year-old patient remarks that she just cannot believe that her breasts "sag so much." She states it must be from a lack of exercise. What explanation should the nurse offer her? a. After menopause, only women with large breasts experience sagging. b. After menopause, sagging is usually due to decreased muscle mass within the breast. c. After menopause, a diet that is high in protein will help maintain muscle mass, which keeps the breasts from sagging. d. After menopause, the glandular and fat tissue atrophies, causing breast size and elasticity to diminish, resulting in breasts that sag.

ANS: D After menopause, the glandular tissue atrophies and is replaced with connective tissue. The fat envelope also atrophies, beginning in the middle years and becoming significant in the eighth and ninth decades of life. These changes decrease breast size and elasticity; consequently, the breasts droop and sag, looking flattened and flabby. There is no or little muscle mass in the breast, so sagging is not due to decreased muscle mass and a diet high in protein will not help minimize sagging.

The nurse is teaching a pregnant woman about breast milk. Which statement by the nurse is correct? a. "Your breast milk is immediately present after the delivery of your baby." b. "Breast milk is rich in protein and sugars (lactose) but has very little fat." c. "The colostrum, which is present right after birth, does not contain the same nutrients as breast milk." d. "You may notice a thick, yellow fluid expressed from your breasts as early as the fourth month of pregnancy."

ANS: D After the fourth month, colostrum may be expressed. This thick yellow fluid is the precursor of milk, and it contains the same amount of protein and lactose but practically no fat. The breasts produce colostrum for the first few days after delivery. It is rich with antibodies that protect the newborn against infection; therefore, breastfeeding is important.

Which of these statements is true regarding the penis? a. The urethral meatus is located on the ventral side of the penis. b. The prepuce is the fold of foreskin covering the shaft of the penis. c. The penis is made up of two cylindric columns of erectile tissue. d. The corpus spongiosum expands into a cone of erectile tissue called the glans.

ANS: D At the distal end of the shaft, the corpus spongiosum expands into a cone of erectile tissue, the glans. The penis is made up of three (not two) cylindric columns of erectile tissue. The prepuce is the skin that covers the glans (not the shaft) of the penis. The urethral meatus forms at the tip of the glans (not on the ventral side).

The nurse should use which test to check for large amounts of fluid around the patella? a. Tinel sign b. Phalen test c. McMurray test d. Ballottement

ANS: D Ballottement of the patella is reliable when large amounts of fluid are present. The Tinel sign and the Phalen test are used to check for carpal tunnel syndrome. The McMurray test is used to test the knee for a torn meniscus.

A patient who is visiting the clinic reports having "stomach pains for 2 weeks" and describes his stools as being "soft and black" for approximately the last 10 days. He denies taking any medications. What do these symptoms suggest? a. Excessive fat caused by malabsorption b. Absent bile pigment from liver problems c. Increased iron intake, resulting from a change in diet d. Occult blood, resulting from gastrointestinal bleeding

ANS: D Black stools may be tarry as a result of occult blood (melena) from gastrointestinal bleeding or nontarry from ingestion of iron medications (not diet). Excessive fat causes the stool to become frothy. The absence of bile pigment causes clay-colored stools

A 35-year-old woman is at the clinic for a gynecologic examination. During the examination, she asks the nurse, "How often do I need to have this Pap test done?" Which reply by the nurse is correct? a. "It depends. Do you smoke?" b. "A Pap test needs to be performed annually until you are 65 years of age." c. "If you have two consecutive normal Pap tests, then you can wait 5 years between tests." d. "After age 30 years, if you have three consecutive normal Pap tests, then you may be screened every 2 to 3 years."

ANS: D Cervical cancer screening with the Pap test continues annually until age 30 years. After age 21, regardless of sexual history or activity, women should be screened every 3 years until age 30, then every 5 years until age 65.

The nurse notices that a woman in an exercise class is unable to do one-person jump rope. What does the nurse know that the shoulder must be able to do in order for one to be able to do one-person jump rope? a. Inversion b. Supination c. Protraction d. Circumduction

ANS: D Circumduction is defined as moving the arm in a circle around the shoulder. This movement is necessary to perform one-person jump rope. Inversion is the moving of the sole of the foot inward at the ankle. Supination is turning the forearm so the palm is down. Protraction is moving a body part forward and parallel to the ground.

When assessing muscle strength, the nurse observes that a patient has complete range of motion against gravity with full resistance. What grade of muscle strength should the nurse record using a 0- to 5-point scale? a. 2 b. 3 c. 4 d. 5

ANS: D Complete range of motion against gravity is normal muscle strength and is recorded as grade 5 muscle strength. The other options are not correct.

The nurse is assessing the neurologic status of a patient who has a late-stage brain tumor. With the reflex hammer, the nurse draws a light stroke up the lateral side of the sole of the foot and inward, across the ball of the foot. In response, the patient's toes fan out, and the big toe shows dorsiflexion. How should the nurse interpret these findings? a. Clonus, which is a hyperactive response b. Achilles reflex, which is an expected response c. Negative Babinski sign, which is normal for adults d. Positive Babinski sign, which is abnormal for adults

ANS: D Dorsiflexion of the big toe and fanning of all toes is a positive Babinski sign, also called up-going toes. This response occurs with upper motor neuron disease of the corticospinal (or pyramidal) tract and is an abnormal finding for adults.

A man who was found wandering in a park at 2 AM has been brought to the emergency department for an examination; he said he fell and hit his head. During the examination, the nurse asks him to use his index finger to touch the nurse's finger, then his own nose, then the nurse's finger again (which has been moved to a different location). The patient is clumsy, unable to follow the instructions, and overshoots the mark, missing the finger. What does the nurse suspect? a. Cerebral injury b. Peripheral neuropathy c. Cerebrovascular accident d. Acute alcohol intoxication

ANS: D During the finger-to-finger test, if the person has clumsy movement with overshooting the mark, either a cerebellar disorder or acute alcohol intoxication should be suspected. The person's movements should be smooth and accurate. The other options are not correct.

During an interview, a patient reveals that she is pregnant. She states that she is not sure whether she will breastfeed her baby and asks for some information about this. Which of these statements by the nurse is accurate? a. "Breastfed babies tend to be more colicky." b. "Breastfed babies eat more often than infants on formula." c. "Breastfeeding is second nature, and every woman can do it." d. "Breastfeeding provides the perfect food and antibodies for your baby."

ANS: D Exclusively breastfeeding for 6 months provides the perfect food and antibodies for the baby, decreases the risk for ear infections, promotes bonding, and provides relaxation. The other statements are not accurate.

What are the fibrous bands that run directly from one bone to another, strengthen the joint, and help prevent movement in undesirable directions called? a. Bursa b. Tendons c. Cartilage d. Ligaments

ANS: D Fibrous bands running directly from one bone to another that strengthen the joint and help prevent movement in undesirable directions are called ligaments. The other options are not correct.

A mother brings her newborn baby boy in for a checkup; she tells the nurse that he does not seem to be moving his right arm as much as his left and that he seems to have pain when she lifts him up under the arms. The nurse suspects a fractured clavicle. What finding would support this suspicion? a. Negative Allis test b. Positive Ortolani sign c. Limited range of motion during Lasègue test d. Limited range of motion during the Moro reflex

ANS: D For a fractured clavicle, the nurse should observe for limited arm range of motion and unilateral response to the Moro reflex. The Allis test and Ortolani sign are performed to assess for hip dislocations, not fractured clavicle. The Lasègue test is performed to assess for sciatica or herniated nucleus pulposus. For a fractured clavicle, the nurse should observe for limited arm range of motion and unilateral response to the Moro reflex. The other tests are not appropriate for this type of fracture.

A 22-year-old woman is being seen at the clinic for problems with vulvar pain, dysuria, and fever. On physical examination, the nurse notices clusters of small, shallow vesicles with surrounding erythema on the labia. Inguinal lymphadenopathy is also present. What do these findings indicate? a. HPV b. Pediculosis pubis c. Contact dermatitis d. Herpes simplex virus type 2

ANS: D Herpes simplex virus type 2 exhibits clusters of small, shallow vesicles with surrounding erythema that erupt on the genital areas. Inguinal lymphadenopathy is also present. The woman reports local pain, dysuria, and fever. HPV presents with pink or flesh-colored, soft, points, moist, painless warty papules on the external genitalia. Pediculosis pubis presents with severe perineal itching and excoriations and erythematous areas on the external genitalia. May see little dark spots (lice are small), nits (eggs), or lice adherent to pubic hair near roots. Contact dermatitis presents as red, swollen vesicles with severe itching that may be a result from contact with an allergenic substance. There may be weeping lesions, crusts, scales, thickening of ski, and excoriations from scratching.

During a health history, a patient tells the nurse that he has trouble in starting his urine stream. How should the nurse document this finding? a. Urgency b. Dribbling c. Frequency d. Hesitancy

ANS: D Hesitancy is trouble in starting the urine stream. Urgency is the feeling that one cannot wait to urinate. Dribbling is the last of the urine before or after the main act of urination. Frequency is urinating more often than usual.

In assessing a 70-year-old patient who has had a recent cerebrovascular accident, the nurse notices right-sided weakness. What might the nurse expect to find when testing his reflexes on the right side? a. Normal reflexes b. Lack of reflexes c. Diminished reflexes d. Hyperactive reflexes

ANS: D Hyperreflexia is the exaggerated reflex observed when the monosynaptic reflex arc is released from the influence of higher cortical levels. This response occurs with upper motor neuron lesions (e.g., a cerebrovascular accident). The other responses are incorrect.

Which of these clinical situations would the nurse consider to be outside normal limits? a. A patient has had one pregnancy and states that she believes she may be entering menopause. Her breast examination reveals breasts that are soft and slightly sagging. b. A patient has never been pregnant. Her breast examination reveals large pendulous breasts that have a firm, transverse ridge along the lower quadrant in both breasts. c. A patient has never been pregnant and reports that she should begin her period tomorrow. Her breast examination reveals breast tissue that is nodular and somewhat engorged. She states that the examination was slightly painful. d. A patient has had two pregnancies, and she breastfed both of her children. Her youngest child is now 10 years old. Her breast examination reveals breast tissue that is somewhat soft, and she has a small amount of thick yellow discharge from both nipples.

ANS: D If any discharge appears, the nurse should note its color and consistency. Except in pregnancy and lactation, any discharge is abnormal. In nulliparous women, normal breast tissue feels firm, smooth, and elastic; after pregnancy, the tissue feels soft and loose. Premenstrual engorgement is normal, and consists of a slight enlargement, tenderness to palpation, and a generalized nodularity. A firm, transverse ridge of compressed tissue in the lower quadrants, known as the inframammary ridge, is especially noticeable in large breasts.

The nurse is performing a genital examination on a male patient and notices urethral drainage. What should the nurse do when collecting urethral discharge for microscopic examination and culture? a. Ask the patient to urinate into a sterile cup. b. Ask the patient to obtain a specimen of semen. c. Insert a cotton-tipped applicator into the urethra. d. Compress the glans between the examiner's thumb and forefinger, and collect any discharge.

ANS: D If urethral discharge is noticed, then the examiner should collect a smear for microscopic examination and culture by compressing the glans anteroposteriorly between the thumb and forefinger. The other options are not correct actions

A nurse is assessing a patient's risk for contracting a sexually transmitted infection (STI). What is an appropriate question to ask this patient? a. "Do you have a sexually transmitted infection?" b. "You are aware of the dangers of unprotected sex, aren't you?" c. "You know that it's important to use condoms for protection, right?" d. "Do you use a condom with each episode of sexual intercourse?

ANS: D In reviewing a patient's risk for STIs, the nurse should ask in a nonconfrontational manner whether condoms are being used during each episode of sexual intercourse. Asking a person whether he or she has an infection does not address the risk.

A 65-year-old woman is in the office for routine gynecologic care. She had a complete hysterectomy 3 months ago after cervical cancer was detected. Which statement does the nurse know to be true regarding this visit? a. Her cervical mucosa will be red and dry looking. b. She will not need to have a Pap smear performed. c. The nurse can expect to find that her uterus will be somewhat enlarged and her ovaries small and hard. d. The nurse should plan to lubricate the instruments and the examining hand adequately to avoid a painful examination.

ANS: D In the aging adult woman, natural lubrication is decreased; therefore, to avoid a painful examination, the nurse should take care to lubricate the instruments and the examining hand adequately. Menopause, with the resulting decrease in estrogen production, shows numerous physical changes. The cervix shrinks and looks pale and glistening. With the bimanual examination, the uterus feels smaller and firmer and the ovaries are not normally palpable. Women should continue cervical cancer screening up to age 65 years if they have an intact cervix and are in good health. Women who have had a total hysterectomy do not need cervical cancer screening if they have 3 consecutive negative Pap tests or 2 or more consecutive negative HIV and Pap tests within the last 10 years.

The nurse is explaining to a patient that there are shock absorbers in his back to cushion the spine and to help it move. What is the nurse referring to as shock absorbers? a. Vertebral column b. Nucleus pulposus c. Vertebral foramen d. Intervertebral disks

ANS: D Intervertebral disks are elastic fibrocartilaginous plates that cushion the spine similar to shock absorbers and help it move. The vertebral column is the spinal column itself. The nucleus pulposus is located in the center of each disk. The vertebral foramen is the channel, or opening, for the spinal cord in the vertebrae.

The nurse is aware that which statement is true regarding the incidence of testicular cancer? a. The cure rate for testicular cancer is low. b. Testicular cancer is the most common cancer in men aged 30 to 50 years. c. The early symptoms of testicular cancer are pain and induration. d. Men with a history of cryptorchidism are at the greatest risk for the development of testicular cancer.

ANS: D Men with undescended testicles (cryptorchidism) are at the greatest risk for the development of testicular cancer. The overall incidence of testicular cancer is rare. Although testicular cancer has no early symptoms, when detected early and treated before metastasizing, the cure rate is almost 100%.

When performing the bimanual examination, the nurse notices that the cervix feels smooth and firm, is round, and is fixed in place (does not move). When cervical palpation is performed, the patient complains of some pain. How should the nurse interpret these findings? a. These findings are all within normal limits. b. Pain may occur during palpation of the cervix. c. Cervical consistency should be soft and velvety—not firm. d. The cervix should move when palpated; an immobile cervix may indicate malignancy.

ANS: D Normally the cervix feels smooth and firm, similar to the consistency of the tip of the nose. It softens and feels velvety at 5 to 6 weeks of pregnancy (Goodell sign). The cervix should be evenly rounded. With a finger on either side, the examiner should be able to move the cervix gently from side to side, and doing so should produce no pain for the patient. Hardness of the cervix may occur with malignancy. Immobility may occur with malignancy, and pain may occur with inflammation or ectopic pregnancy

A 22-year-old woman has been considering using oral contraceptives. As a part of her health history, what should the nurse ask? a. "Do you have a history of heart murmurs?" b. "Will you be in a monogamous relationship?" c. "Have you carefully thought this choice through?" d. "If you smoke, how many cigarettes do you smoke per day?"

ANS: D Oral contraceptives, together with cigarette smoking, increase the risk for cardiovascular side effects. If cigarettes are used, then the nurse should assess the patient's smoking history. The other questions are not appropriate.

The nurse is providing patient teaching about an erectile dysfunction drug. One of the drug's potential side effects is prolonged, painful erection of the penis without sexual stimulation. What is the medical term for this condition? a. Orchitis b. Phimosis c. Stricture d. Priapism

ANS: D Priapism is prolonged, painful erection of the penis without sexual desire. Orchitis is inflammation of the testes. Stricture is a narrowing of the opening of the urethral meatus. Phimosis is the inability to retract the foreskin.

A woman who is 8 weeks pregnant is in the clinic for a checkup. The nurse reads on her chart that her cervix is softened and looks cyanotic. Based on these findings, what two signs is the patient exhibiting? a. Tanner and Hegar b. Hegar and Goodell c. Chadwick and Hegar d. Goodell and Chadwick

ANS: D Shortly after the first missed menstrual period, the female genitalia show signs of the growing fetus. The cervix softens (Goodell sign) at 4 to 6 weeks, and the vaginal mucosa and cervix look cyanotic (Chadwick sign) at 8 to 12 weeks. These changes occur because of increased vascularity and edema of the cervix and hypertrophy and hyperplasia of the cervical glands. Hegar sign occurs when the isthmus of the uterus softens at 6 to 8 weeks. Tanner sign is not a correct response.

A patient is unable to perform rapid alternating movements such as rapidly patting her knees. How should the nurse document this finding? a. Ataxia b. Astereognosis c. Loss of kinesthesia d. Presence of dysdiadochokinesia

ANS: D Slow clumsy movements and the inability to perform rapid alternating movements occur with cerebellar disease. The condition is termed dysdiadochokinesia. Ataxia is an uncoordinated or unsteady gait. Astereognosis is the inability to identify an object by feeling it. Kinesthesia is the person's ability to perceive passive movement of the extremities or the loss of position sense.

A 52-year-old patient states that when she sneezes or coughs she "wets herself a little." She is very concerned that something may be wrong with her. What does this finding suggest? a. Dysuria b. Hematuria c. Urge incontinence d. Stress incontinence

ANS: D Stress incontinence is involuntary urine loss with physical strain, sneezing, or coughing. Dysuria is pain or burning with urination. Hematuria is bleeding with urination. Urge incontinence is involuntary urine loss that occurs as a result of an overactive detrusor muscle in the bladder that contracts and causes an urgent need to void.

A 68-year-old woman has come in for an assessment of her rheumatoid arthritis, and the nurse notices raised, firm, nontender nodules at the olecranon bursa and along the ulna. What is the appropriate term for these nodules? a. Epicondylitis b. Gouty arthritis c. Olecranon bursitis d. Subcutaneous nodules.

ANS: D Subcutaneous nodules are raised, firm, and nontender and occur with rheumatoid arthritis in the olecranon bursa and along the extensor surface of the ulna. Epicondylitis (Tennis elbow) is pain at the lateral epicondyle of the humerus. Gout is a painful inflammatory arthritis characterized by excess uric acid in the blood and deposits of urate crystals in the joint space. Symptoms include redness, swelling, heat and extreme pain. Olecranon bursitis is a large, soft knob or "goose egg" and redness from swelling and inflammation of olecranon bursa.

During an examination, the nurse notices that a male patient has a red, round, superficial ulcer with a yellowish serous discharge on his penis. On palpation, the nurse finds a nontender base that feels like a small button between the thumb and fingers. What do these findings indicate? a. Genital warts b. Herpes infection c. Carcinoma lesion d. Syphilitic chancre

ANS: D Syphilitic chancres begin as a small, solitary, silvery papule within 2 to 4 weeks of infection which then erodes to a red, round or oval, superficial ulcer with a yellowish serous discharge. The lesions of genital warts are soft, pointed, moist, fleshy, painless papules that may be single or multiple in a cauliflower-like patch. They occur on the shaft of the penis, behind the corona, or around the anus, where they may grow into large grapelike clusters. Genital herpes (HSV-2 infection) appears as clusters of small vesicles with surrounding erythema which are often painful and erupt on the glans, foreskin, or anus. Genital carcinoma begins as red, raised, warty growth or as an ulcer with watery discharge which almost always occur on the glans or inner lip of foreskin. The symptoms this patient is experiencing are those of syphilitic chancre.

START OF FEMALE GENITOURINARY SYSTEM QUESTIONS During a health history, a 22-year-old woman asks, "Can I get that vaccine for human papilloma virus (HPV)? I have genital warts and I'd like them to go away!" What is the nurse's best response? a. "The HPV vaccine is for girls and women ages 9 to 26 years, so we can start that today." b. "This vaccine is only for girls who have not yet started to become sexually active." c. "Let's check with the physician to see if you are a candidate for this vaccine." d. "The vaccine cannot protect you if you already have an HPV infection."

ANS: D The HPV vaccine is appropriate for girls and women age 9 to 26 years and is administered to prevent cervical cancer by preventing HPV infections before girls become sexually active. However, it cannot protect the woman if an HPV infection is already present

START OF ANUS, RECTUM, PROSTATE QUESTIONS Which statement concerning the anal canal is true? a. Slants backward toward the sacrum b. Contains hair and sebaceous glands c. Approximately 2 cm long in the adult d. The outlet for the gastrointestinal tract

ANS: D The anal canal is the outlet for the gastrointestinal tract and is approximately 3.8 cm long in the adult. It is lined with a modified skin that does not contain hair or sebaceous glands, and it slants forward toward the umbilicus.

While performing an assessment of the perianal area of a patient, the nurse notices that the pigmentation of the anus is darker than the surrounding skin, the anal opening is closed, and a skin sac that is shiny and blue is noted. The patient mentioned that he has had pain with bowel movements and has occasionally noted some spots of blood. What does this assessment and history most likely indicate? a. Anal fistula b. Pilonidal cyst c. Rectal prolapse d. Thrombosed hemorrhoid

ANS: D The anus normally looks moist and hairless, with coarse folded skin that is more pigmented than the perianal skin, and the anal opening is tightly closed. The shiny blue skin sac indicates a thrombosed hemorrhoid.

During an internal examination, the nurse notices that the cervix bulges outside the introitus when the patient is asked to strain. How should the nurse document this finding? a. A normal finding b. Uterine prolapse, graded first degree c. Uterine prolapse, graded third degree d. Uterine prolapse, graded second degree

ANS: D The cervix should not be found to bulge into the vagina. Uterine prolapse is graded as follows: first degree—the cervix appears at the introitus with straining; second degree—the cervix bulges outside the introitus with straining; and third degree—the whole uterus protrudes, even without straining (essentially, the uterus is inside out)

A professional tennis player comes into the clinic complaining of a sore elbow. Where should the nurse assess for tenderness? a. Olecranon bursa b. Annular ligament c. Base of the radius d. Medial and lateral epicondyle

ANS: D The epicondyles, the head of the radius, and the tendons are common sites of inflammation and local tenderness, commonly referred to as tennis elbow. The other locations are not affected.

The nurse is testing the function of CN XI. Which statement best describes the response the nurse should expect if this nerve is intact? a. Patient demonstrates the ability to hear normal conversation. b. When patient sticks out tongue it is midline and without tremors or deviation. c. Patient follows an object with his or her eyes without nystagmus or strabismus. d. Patient moves the head and shoulders against resistance with equal strength.

ANS: D The following normal findings are expected when testing the spinal accessory nerve (CN XI): The patient's sternomastoid and trapezius muscles are equal in size; the person can forcibly rotate the head both ways against resistance applied to the side of the chin with equal strength; and the patient can shrug the shoulders against resistance with equal strength on both sides. Checking the patient's ability to hear normal conversation checks the function of CN VIII. Having the patient stick out the tongue checks the function of CN XII. Testing the eyes for nystagmus or strabismus is performed to check CNs III, IV, and VI.

During the assessment of an 18-month-old infant, the mother expresses concern to the nurse about the infant's inability to toilet train. What would be the best response by the nurse? a. "Some children are just more difficult to train, so I wouldn't worry about it yet." b. "Have you considered reading any of the books on toilet training? They can be very helpful." c. "This could mean that there is a problem with your baby's development. We'll watch her closely for the next few months." d. "The nerves that allow your baby to have control over the passing of stools are not developed until at least 18 to 24 months of age."

ANS: D The infant passes stools by reflex. Voluntary control of the external anal sphincter cannot occur until the nerves supplying the area have become fully myelinated, usually around 1 1/2 to 2 years of age. Toilet training usually starts after the age of 2 years.

When the nurse is conducting a sexual history from a male adolescent, which statement would be most appropriate to use at the beginning of the interview? a. "Do you use condoms?" b. "You don't masturbate, do you?" c. "Have you had sex in the last 6 months?" d. "Often adolescents your age have questions about sexual activity."

ANS: D The interview should begin with a permission statement, which conveys that it is normal and acceptable to think or feel a certain way. Sounding judgmental, such as saying "You don't masturbate, do you?" should be avoided.

A patient is visiting the clinic for an evaluation of a swollen, painful knuckle. The nurse notices that the knuckle above his ring on the left hand is swollen and that he is unable to remove his wedding ring. What is the name of this patient's affected joint? a. Tibiotalar b. Interphalangeal c. Tarsometatarsal d. Metacarpophalangeal

ANS: D The joint located just above the ring on the finger is the metacarpophalangeal joint. The interphalangeal joint is located distal to the metacarpophalangeal joint. The tarsometatarsal and tibiotalar joints are found in the foot and ankle.

When reviewing the musculoskeletal system, the nurse should recall that hematopoiesis takes place where? a. Liver b. Spleen c. Kidneys d. Bone marrow

ANS: D The musculoskeletal system functions to encase and protect the inner vital organs, to support the body, to produce red blood cells (hematopoiesis) in the bone marrow, and to store minerals. The other options are not correct. The liver has many functions such as detoxifying the blood, production of bile, and synthesis of proteins needed for blood to clot, but hematopoiesis is not one of its functions. The spleen has many functions such as filtering the blood as part of the immune system, recycling old red blood cells, and storing platelets and white bloods cells but it is not the location of hematopoiesis. The kidney also has many functions such as maintaining fluid balance, filtering minerals, and production of hormones that help stimulate red blood cells production; however, it is not the location of hematopoiesis.

When the nurse is testing the triceps reflex, what is the expected response? a. Flexion of the hand b. Pronation of the hand c. Flexion of the forearm d. Extension of the forearm

ANS: D The normal response of the triceps reflex is extension of the forearm. The normal response of the biceps reflex causes flexion of the forearm. The other responses are incorrect.

Which statement would be most appropriate when the nurse is introducing the topic of sexual relationships during a health interview? a. "Now, it is time to talk about your sexual history. When did you first have intercourse?" b. "Most women your age have had more than one sexual partner. How many would you say you have had?" c. "Women often feel dissatisfied with their sexual relationships. Would it be okay to discuss this now?" d. "Women often have questions about their sexual relationship and how it affects their health. Do you have any questions?"

ANS: D The nurse should begin with an open-ended question to assess individual needs. The nurse should include appropriate questions as a routine part of the health history, because doing so communicates that the nurse accepts the individual's sexual activity and believes it is important. The nurse's comfort with the discussion prompts the patient's interest and, possibly, relief that the topic has been introduced. The initial discussion establishes a database for comparison with any future sexual activities and provides an opportunity to screen sexual problems.

A 13-year-old girl is visiting the clinic for a sports physical examination. The nurse should remember to include which of these tests in the examination? a. Valsalva maneuver b. Testing for occult blood c. Internal palpation of the anus d. Inspection of the perianal area

ANS: D The perianal region of the school-aged child and adolescent should be inspected during the examination of the genitalia. Internal palpation is not routinely performed at this age. Testing for occult blood and performing the Valsalva maneuver are also not necessary.

A 60-year-old man has just been told that he has benign prostatic hypertrophy (BPH). He has a friend who just died from cancer of the prostate and is concerned this will happen to him. How should the nurse respond? a. "The swelling in your prostate is only temporary and will go away." b. "We will treat you with chemotherapy so we can control the cancer." c. "It would be very unusual for a man your age to have cancer of the prostate." d. "The enlargement of your prostate is caused by hormonal changes, and not cancer."

ANS: D The prostate gland commonly starts to enlarge during the middle adult years. BPH is present in 1 in 10 men at the age of 40 years and increases with age. It is believed that the hypertrophy is caused by hormonal imbalance that leads to the proliferation of benign adenomas. The other responses are not appropriate.

During a health history interview, a female patient states that she has noticed a few drops of clear discharge from her right nipple. What should the nurse do next? a. Ask her if she is possibly pregnant. b. Immediately contact the physician to report the discharge. c. Immediately obtain a sample for culture and sensitivity testing. d. Ask the patient some additional questions about the medications she is taking.

ANS: D The use of some medications, such as oral contraceptives, phenothiazines, diuretics, digitalis, steroids, methyldopa, and calcium channel blockers, may cause clear nipple discharge. Bloody or blood-tinged discharge from the nipple, not clear, is significant, especially if a lump is also present. In the pregnant female, colostrum may be expressed after the fourth month of pregnancy, but colostrum would be a thick, yellowish liquid, not clear. In the pregnant female, colostrum would be a thick, yellowish liquid, and it would be normally expressed after the fourth month of pregnancy.

The nurse is reviewing risk factors for breast cancer. Which of these women have risk factors that place them at a higher risk for breast cancer? a. 37-year-old who is slightly overweight b. 42-year-old who has had ovarian cancer c. 45-year-old who has never been pregnant d. 66-year-old whose mother had breast cancer

ANS: D The woman at highest risk is the 66-year-old woman whose mother had breast cancer. This woman has two risk factors for breast cancer with >4.0 relative risk, which are her age (≥65) and a first-degree relative (her mother) with breast cancer. The 37-year-old woman who is slightly overweight does not have any risk factors for breast cancer. The 42-year-old woman who has had ovarian cancer has one risk factor for breast cancer with a relative risk for 1.1 to 2.0 and that is her personal history of ovarian cancer. The 45-year-old woman who has never been pregnant has one risk factor for breast cancer with a relative risk for 1.1 to 2.0 and that is having no full-term pregnancies.

The nurse is performing a breast examination. Which of these statements best describes the correct procedure to use when screening for nipple and skin retraction during a breast examination? a. Have the patient bend over and touch her toes. b. Have the patient lie down on her left side and observe for any retraction. c. Have the patient shift from a supine position to a standing position, and note any lag or retraction. d. Have the patient slowly lift her arms above her head, and note any retraction or lag in movement.

ANS: D The woman should be directed to change position while checking the breasts for signs of skin retraction. Initially, she should be asked to lift her arms slowly over her head. Both breasts should move up symmetrically. Retraction signs are due to fibrosis in the breast tissue, usually caused by growing neoplasms. The nurse should notice whether movement of one breast is lagging.

During an assessment of a 32-year-old patient with a recent head injury, the nurse notices that the patient responds to pain by extending, adducting, and internally rotating his arms. His palms pronate, and his lower extremities extend with plantar flexion. Which statement concerning these findings is most accurate? What do these findings indicate? a. A lesion of the cerebral cortex b. A completely nonfunctional brainstem c. Normal findings that will resolve in 24 to 48 hours d. A very ominous sign and may indicate brainstem injury

ANS: D These findings are all indicative of decerebrate rigidity, which is a very ominous condition and may indicate a brainstem injury.

A 25-year-old woman comes to the emergency department with a sudden fever of 38.3° C and abdominal pain. Upon examination, the nurse notices that she has rigid, boardlike lower abdominal musculature. When the nurse tries to perform a vaginal examination, the patient has severe pain when the uterus and cervix are moved. What do these findings suggest? a. Endometriosis b. Uterine fibroids c. Ectopic pregnancy d. Pelvic inflammatory disease

ANS: D These signs and symptoms are suggestive of acute pelvic inflammatory disease, also known as acute salpingitis. Endometriosis may have masses (in various locations in the pelvic area) that are small, firm, nodular, and tender to palpation, with enlarged ovaries. Uterine fibroids often are asymptomatic. Symptoms that may occur included vague discomfort, bloating, heaviness, pelvic pressure, dyspareunia, urinary frequency backache or excessive uterine bleeding. The uterus may be irregularly enlarged, firm, mobile and nodular with hard, painless nodules in the uterine wall. An ectopic pregnancy presents with sharp, stabbing abdominal or pelvic pain, vaginal spotting or new-onset bleeding, and positive pregnancy test. There will likely be a softening of the cervix and fundus; movement of cervix and uterus causes pain; and palpable tender, round mobile swelling, lateral to uterus.

As the nurse is taking the health history, the patient states, "It really hurts back there, and sometimes it itches, too. I have even seen blood on the tissue when I have a bowel movement. Is there something there?" The nurse should expect to see which of these upon examination of the anus? a. Rectal prolapse b. Internal hemorrhoid c. External hemorrhoid that has resolved d. External hemorrhoid that is thrombosed

ANS: D These symptoms are consistent with an external hemorrhoid. An external hemorrhoid, when thrombosed, contains clotted blood and becomes a painful, swollen, shiny blue mass that itches and bleeds with defecation. When the external hemorrhoid resolves, it leaves a flabby, painless skin sac around the anal orifice. An internal hemorrhoid is not palpable but may appear as a red mucosal mass when the person performs a Valsalva maneuver. A rectal prolapse appears as a moist, red doughnut with radiating lines.

When the nurse asks a 68-year-old patient to stand with his feet together and arms at his side with his eyes closed, he starts to sway and moves his feet farther apart. How should the nurse document this finding? a. Ataxia b. Lack of coordination c. Negative Homan sign d. Positive Romberg sign

ANS: D This is an abnormal, or positive, Romberg test. Abnormal findings for the Romberg test include swaying, falling, and a widening base of the feet to avoid falling. A positive Romberg sign is a loss of balance that is increased by the closing of the eyes. Ataxia is an uncoordinated or unsteady gait. Homan sign is used to test the legs for deep-vein thrombosis. Ataxia is an uncoordinated or unsteady gait. Homan sign is used to test the legs for deep-vein thrombosis. These findings are an abnormal, or positive, Romberg test. Abnormal findings for the Romberg test include swaying, falling, and a widening base of the feet to avoid falling

The nurse is examining a 3-month-old infant. While the nurse holds his or her thumbs on the infant's inner mid thighs and the fingers on the outside of the infant's hips, touching the greater trochanter, the nurse adducts the legs until the his or her thumbs touch and then abducts the legs until the infant's knees touch the table. The nurse does not notice any "clunking" sounds. How should the nurse document this finding? a. Positive Allis test b. Negative Allis test c. Positive Ortolani sign d. Negative Ortolani sign

ANS: D This maneuver is the Ortolani sign. Normally this maneuver feels smooth and has no sound (negative Ortolani sign). However, with a positive Ortolani sign the nurse will feel and hear a "clunk," as the head of the femur pops back into place. A positive Ortolani sign also reflects hip instability. The Allis test also tests for hip dislocation but is performed by comparing leg lengths. The Allis test is a test that assesses for hip dislocation but comparing leg lengths. The maneuver described in this question is the Ortolani sign. Normally this maneuver feels smooth and has no sound (negative Ortolani sign). However, with a positive Ortolani sign the nurse will feel and hear a "clunk," as the head of the femur pops back into place. A positive Ortolani sign also reflects hip instability.

A 70-year-old woman tells the nurse that every time she gets up in the morning or after she's been sitting for a while, she gets "really dizzy" and feels like she is going to fall over. What is the best response by the nurse? a. "Have you been extremely tired lately?" b. "You probably just need to drink more liquids." c. "I'll refer you for a complete neurologic examination." d. "You need to get up slowly when you've been lying down or sitting."

ANS: D This patient's symptoms are unlikely r/t being tired or dehydration and would not require a complete neurological examination at this time. Instead, they are likely due to normal aging. Aging is accompanied by a progressive decrease in cerebral blood flow. In some people, this decrease causes dizziness and a loss of balance with a position change. These individuals need to be taught to get up slowly. The other responses are incorrect.

Which statement concerning the testes is true? a. The vas deferens is located along the inferior portion of each testis. b. The lymphatic vessels of the testes drain into the abdominal lymph nodes. c. The right testis is lower than the left because the right spermatic cord is longer. d. The cremaster muscle contracts in response to cold and draws the testicles closer to the body.

ANS: D When it is cold, the cremaster muscle contracts, which raises the scrotal sac and brings the testes closer to the body to absorb heat necessary for sperm viability. The lymphatic vessels of the testes drain into the inguinal lymph nodes. The vas deferens is located along the upper portion of each testis. The left testis is lower than the right because the left spermatic cord is longer.

When the nurse is performing a genital examination on a male patient, which action is correct? a. Auscultating for the presence of a bruit over the scrotum b. Palpating the inguinal canal only if a bulge is present in the inguinal region during inspection c. Palpating for the vertical chain of lymph nodes along the groin, inferior to the inguinal ligament d. Having the patient shift his weight onto the left (unexamined) leg when palpating for a hernia on the right side

ANS: D When palpating for the presence of a hernia on the right side, the male patient is asked to shift his weight onto the left (unexamined) leg. Auscultating for a bruit over the scrotum is not appropriate. When palpating for lymph nodes, the horizontal chain is palpated. The inguinal canal should be palpated whether a bulge is present or not.

When the nurse is performing a genital examination on a male patient, the patient has an erection. How should the nurse respond? a. Ask the patient if he would like someone else to examine him. b. Continue with the examination as though nothing has happened. c. Stop the examination, leave the room while stating that the examination will resume at a later time. d. Reassure the patient that this is a normal response and continue with the examination.

ANS: D When the male patient has an erection, the nurse should reassure the patient that this is a normal physiologic response to touch and proceed with the rest of the examination. The other responses are not correct and may be perceived as judgmental

When performing a genitourinary assessment on a 16-year-old male adolescent, the nurse notices a swelling in the scrotum that increases with increased intra-abdominal pressure and decreases when he is lying down. The patient reports pain when straining. What do these findings indicate? a. Femoral hernia b. Incisional hernia c. Direct inguinal hernia d. Indirect inguinal hernia

ANS: D With indirect inguinal hernias, pain occurs with straining and a soft swelling increases with increased intra-abdominal pressure, which may decrease when the patient lies down. A femoral hernia usually presents with pain that is constant and may be severe and become strangulated. An incisional, or ventral, hernia occurs at the site of a previous surgical incision in either the groin or abdominal area. A direct hernia is usually painless and is easily reduced when supine. The symptoms this patient is experiencing are those of an indirect hernia.

A 46-year-old woman is in the clinic for her annual gynecologic examination. She voices concern about ovarian cancer because her mother and sister died of it. Which statement does the nurse know to be correct regarding ovarian cancer? a. Ovarian cancer rarely has any symptoms. b. The Pap smear detects the presence of ovarian cancer. c. Women over age 40 years should have a thorough pelvic examination every 3 years. d. Women at high risk for ovarian cancer should have annual transvaginal ultrasonography for screening.

ANS: D With ovarian cancer, the patient may have abdominal pain, pelvic pain, increased abdominal size, bloating, and nonspecific gastrointestinal symptoms; or she may be asymptomatic. The Pap smear does not detect the presence of ovarian cancer. Annual transvaginal ultrasonography may detect ovarian cancer at an earlier stage in women who are at high risk for developing it.

A 78-year-old man has a history of a cerebrovascular accident. The nurse notes that when he walks, his left arm is immobile against the body with flexion of the shoulder, elbow, wrist, and fingers and adduction of the shoulder. His left leg is stiff and extended and circumducts with each step. What type of gait disturbance is this individual experiencing? a. Scissors gait b. Cerebellar ataxia c. Parkinsonian gait d. Spastic hemiparesis

ANS: D With spastic hemiparesis, the arm is immobile against the body. Flexion of the shoulder, elbow, wrist, and fingers occurs, and adduction of the shoulder, which does not swing freely, is observed. The leg is stiff and extended and circumducts with each step. Causes of this type of gait include cerebrovascular accident. With scissors gait the knees cross or are in contact, like holding an orange between the thighs, and the person uses short steps, and walking requires effort. The characteristics of cerebellar ataxia include a staggering, wide-based gait; difficulty with turns; and uncoordinated movement with positive Romberg sign. Parkinsonian gait presents with a stooped posture with trunk pitched forward. Elbows, hips and knees are flexed. Steps are short and shuffling. The gait disturbance of this patient is spastic hemiparesis. With spastic hemiparesis, the arm is immobile against the body. Flexion of the shoulder, elbow, wrist, and fingers occurs, and adduction of the shoulder, which does not swing freely, is observed. The leg is stiff and extended and circumducts with each step. Causes of this type of gait include cerebrovascular accident

A 55-year-old man is in the clinic for a yearly checkup. He is worried because his father died of prostate cancer. The nurse knows which tests should be performed at this time? (Select all that apply.) a. Urinalysis b. Prostate biopsy c. Transrectal ultrasound d. Digital rectal examination (DRE) e. Blood test for prostate-specific antigen (PSA)

ANS: D, E Prostate cancer is typically detected by testing the blood for PSA or by a DRE. It is recommended that both PSA and DRE be offered to men annually, beginning at age 50 years. If PSA is elevated, then further laboratory work or a transrectal ultrasound (TRUS) and biopsy may be recommended.

The nurse is palpating an ovarian mass during an internal examination of a 63-year-old woman. Which findings of the mass's characteristics would suggest the presence of an ovarian cyst? (Select all that apply.) a. Fixed b. Poorly defined c. Heavy and solid d. Mobile and solid e. Smooth and round f. Mobile and fluctuant

ANS: E, F An ovarian cyst (fluctuant ovarian mass) is usually asymptomatic and would feel like a smooth, round, fluctuant, mobile, nontender mass on the ovary. A mass that is heavy, solid, fixed, and poorly defined suggests malignancy. A benign mass may feel mobile and solid.


संबंधित स्टडी सेट्स

Microeconomics Exam 01 Study Guide

View Set

Math 4 DWSBA #3 (Topics 9, 10, 11, 12)

View Set

Sociology Ch. 1-16 Questions ... GOOD LUCK

View Set

Music Theory Chapters 9 - 15 Quiz

View Set